Site Loader

Содержание

Тест № 2. Скалярное произведение векторов.

Тест №2

Скалярное произведение векторов

1 Вариант

  1. Найдите угол между векторами и =2 — 3.

а) 0°; б) определить нельзя; в) 45°; г)90°; д)180°.

2. Вектор составляет с положительным направлением оси Ох угол 135°. Найдите абсциссу вектора , если || =2.

а) 2; б)-2; в)-; г); д) определить нельзя.

3. Даны точки А(3; -2; 4), В(4; -1; 2), С(6; -3; 2), D(7; -3; 1). Найдите угол между векторами и .

а) 150°; б)30°; в)45°; г)60°; д)120°.

4. Угол между векторами и равен 60°. Найдите длину вектора 2 — , если ||=4,

||= 2.

а)2; б)10; в)5; г)2; д)2.

5. Выберите верное утверждение.

а) Угол между двумя векторами не может быть тупым;

б) скалярный квадрат вектора равен квадрату его длины;

в) скалярное произведение нулевых векторов равно нулю тогда и только тогда, когда эти векторы перпендикулярны;

г) ненулевой вектор называется направляющим вектором прямой, если он лежит на прямой, перпендикулярной к данной прямой;

д) скалярное произведение векторов {x; y; z} и {m; n; p} выражается формулой =xp+yn+zm.

6. DABC – правильный тетраэдр. Упростите выражение

(+)*(-)+*(-).

а) 2; б)1; в)-1; г) определить нельзя; д) 0.

7.Дан куб ABCDA1B1C1D1 с ребром 2. Вычислите угол между векторами и , если М – центр грани BCC1

B1.

а)arccos ; б)180°-arccos; в)arcsin; г) -arcsin; д)90°.

8. Дан куб ABCDA1B1C1D1 с ребром 2. Вычислите угол между прямыми АВ1 и ВС1.

а) 120°; б)90°; в)60°; г)150°; д)30°.

9. Ребро куба ABCDA1B1C1D1равно 2, М – центр грани ВСС1В1. Вычислите угол между прямой MD и плоскостью ABC.

а) arccos ; б) 180°-arccos; в)-arcsin; г) arcsin; д)90°.

10. Дан куб ABCDA1B1C1D1 с ребром 2. Вычислите расстояние между серединами отрезков АВ1 и ВС1.

а)2; б)0,5; в); г); д)2.

2 Вариант

  1. Найдите угол между векторами и =-2 + .

а) 180°; б) 45°; в) определить нельзя; г)90°; д)0°.

2. Вектор составляет с положительным направлением оси Оy угол 150°. Найдите ординату вектора , если || =2.

а) определить нельзя; б)-; в); г); д) -3.

3. Даны точки А(5; -8; -1),

В(6; -8; -2), С(7; -5; -11), D(7; -7; -9). Найдите угол между векторами и .

а) 120°; б)60°; в)45°; г)30°; д)150°.

4. Угол между единичными векторами и равен 60°. Найдите длину вектора 2 + .

а); б)7; в); г); д).

5. Выберите верное утверждение.

а) Если угол между двумя векторами равен 90°, то векторы называются перпендикулярными;

б) скалярным произведением двух векторов называется произведение их длин на синус угла между ними;

в) скалярный квадрат любого вектора есть число положительное;

г) от перемены мест сомножителей скалярное произведение изменяется;

д) скалярное произведение векторов {x;

y; z} и {m; n; p} выражается формулой =xn+yp+zm.

6. Все ребра пирамиды HPMKE равны. Упростите выражение

(+)*(-) + *(+).

а) 2; б) определить нельзя; в)-1; г) 0; д) 1.

7.Дан куб ABCDA1B1C1D1 с ребром 4. Вычислите угол между векторами и , если М – центр грани ADA1D1.

а)arcsin ; б)180°-arccos; в)arccos; г) -arcsin; д)90°.

8. Дан куб ABCDA1B1C1D1 с ребром 4. Вычислите угол между прямыми

A1B и AD1.

а) 90°; б)120°; в)60°; г)150°; д)30°.

9. Ребро куба ABCDA1B1C1D1равно 4, М – центр грани ADD1A1. Вычислите угол между прямой MB и плоскостью ABC.

а) arccos ; б) 180°-arccos; в)arcsin; г)- arcsin; д)90°.

10. Дан куб ABCDA1B1C1D1 с ребром 4. Вычислите расстояние между серединами отрезков АВ1 и ВС1

.

а)4; б); в)2; г)2; д)4.

Скалярное произведение векторов

Угол между векторами   может принимать значения от 0 до 180 градусов (от 0 до   радиан) включительно. Аналитически данный факт записывается в виде двойного неравенства:   либо   (в радианах).

В литературе значок угла   часто пропускают и пишут просто  .

Определение: Скалярным произведением двух векторов   и   называется ЧИСЛО, равное произведению длин этих векторов на косинус угла между ними:

Обозначение: скалярное произведение обозначается через   или просто  .

Результат операции является ЧИСЛОМ: Умножается вектор на вектор, а получается число. Действительно, если длины векторов   – это числа, косинус угла – число, то их произведение   тоже будет числом.

Пример

Найти скалярное произведение векторов   и  , если 

Решение: 

Ответ: 

Угол между векторами и значение скалярного произведения

Длины ненулевых векторов всегда положительны:  , поэтому знак может зависеть только от значения косинуса.

1) Если угол между векторами острый:    (от 0 до 90 градусов), то  , и скалярное произведение будет положительным:  . Особый случай: если векторы сонаправлены, то угол между ними считается нулевым  , и скалярное произведение также будет положительным. Поскольку  , то формула упрощается:  .

2) Если угол между векторами тупой:    (от 90 до 180 градусов), то  , и, соответственно, скалярное произведение отрицательно:  . Особый случай: если векторы направлены противоположно, то угол между ними считается развёрнутым:   (180 градусов). Скалярное произведение тоже отрицательно, так как 

3) Если угол между векторами прямой:   (90 градусов), то   и скалярное произведение равно нулю:  .

Скалярное произведение двух векторов равно нулю тогда и только тогда, когда данные векторы ортогональны. Короткая математическая запись: 

Третий случай имеет большую практическую значимость, поскольку позволяет проверить, ортогональны векторы или нет.

Скалярный квадрат вектора Что будет, если вектор умножить на самого себя?

Или: 

Число   называется скалярным квадратом вектора  , и обозначатся как  .

Таким образом, скалярный квадрат вектора   равен квадрату длины данного вектора:

Из данного равенства можно получить формулу для вычисления длины вектора:

Свойства скалярного произведения.

Для произвольных векторов   и любого числа   справедливы следующие свойства:

1)   – переместительный или коммутативный закон скалярного произведения.

2)   – распределительный или 

дистрибутивный закон скалярного произведения. Попросту, можно раскрывать скобки.

3)   – сочетательный или ассоциативный закон скалярного произведения. Константу можно вынести из скалярного произведения.

Пример

Найти скалярное произведение векторов   и  , если известно, что  .

Решение: 

(1) Подставляем выражения векторов  .

(2) Раскрываем скобки по правилу умножения многочленов. Раскрыть скобки нам позволяет дистрибутивное свойство скалярного произведения.

(3) В первом и последнем слагаемом компактно записываем скалярные квадраты векторов:  . Во втором слагаемом используем перестановочность скалярного произведения:  .

(4) Приводим подобные слагаемые:  .

(5) В первом слагаемом используем формулу скалярного квадрата  , о которой не так давно упоминалось. В последнем слагаемом, соответственно, работает та же штука:  . Второе слагаемое раскладываем по стандартной формуле  .

(6) Подставляем данные условия  , и ВНИМАТЕЛЬНО проводим окончательные вычисления.

Ответ: 

Пример

Найти длину вектора  , если  .

Решение:

 

(1) Поставляем выражение вектора  .

(2) Используем формулу длины:  , при этом в качестве вектора «вэ» у нас выступает целое выражение  .

(3) Используем школьную формулу квадрата суммы  .

(4) Дальнейшее аналогично действиям из двух предыдущих задач.

Ответ: 

Осторожно, Вектор!

Осторожно, Вектор!

Автор статьи — Чернов Н.С.

При изучении Физики приходится иметь дело с векторными величинами. Знание основ векторной алгебры является не только необходимым условием успешного и оптимального решения практических задач, но подчас и фундаментом для правильного толкования многих физических понятий.

К сожалению (и удивлению) приходится констатировать, что подавляющее большинство учебников и учебных пособий уделяют недостаточно внимания этому важнейшему вопросу, хотя многие книги по физике и начинаются с изучения основ векторной алгебры.

Это проявляется, во-первых, в неполном охвате ее основных положений, во-вторых, в необоснованном усложнении (либо, наоборот, в упрощении) некоторых понятий, и, в-третьих, в непоследовательном применении или вообще не использовании ранее данных понятий. На что, на мой взгляд, следует обратить внимание учителей?

Важнейшим условием успешного преподавания Физики является обязательное предварительное изучение основ векторной алгебры. Без всякого преувеличения следует еще раз подчеркнуть: от этого будет зависеть успех всего дела! Не следует жалеть затраченных на это изучение часов: эти часы в дальнейшем «возвратятся» в результате того, что с применением методов векторного анализа, некоторые вопросы потребуют меньшего времени на их изучение. Экономится также время, затраченное учениками на повторение пройденного материала. Но самое главное заключается в том, что учащиеся осваивают единый мощнейший метод изучения тех разделов и тем всех без исключения разделов физики, где встречаются векторные величины (механика, электростатика, электродинамика, магнетизм).

Какие же основные понятия и действия над векторными величинами учащиеся должны знать перед изучением основного курса? Перечислю их.

  1. Понятие о векторных и скалярных величинах.
  2. Сложение векторов.
  3. Вычитание векторов.
  4. Умножение вектора на скаляр.
  5. Умножение векторов: а) скалярное умножение, б) векторное умножение.
  6. Проекции векторов: а) скалярная проекция, б) векторная проекция.
  7. Нахождение модуля вектора по его проекциям.

Кратко остановимся на каждом из этих пунктов.

Понятие о векторных и скалярных величинах.

Об определении скалярных величин. Часто говорят, что скаляры – это величины, для характеристики которых достаточно указания только числовой меры, то есть это такие свойства изучаемых объектов, которые полностью характеризуются числом (положительным или отрицательным). Возможно, такого определения будет достаточно, учитывая, что определение скалярных величин дается, в основном для их, если так можно сказать, «противопоставления» векторам, для того, чтобы сделать акцент на главном свойстве векторной величины – направлении. Однако преподаватель должен знать, что такое определение скаляров является неполным. Нужно еще указать, что числовое значение скалярной величины не зависит от выбора системы координат и начала отсчета времени.

В соответствии с этим определением не являются скалярными величины, использованные в следующих предложениях: «Санкт-Петербург расположен на 30-м градусе восточной долготы», «Знаменитые «Начала» Ньютон опубликовал в 1687 году», «Сейчас 12 часов по Московскому времени», «Точка А имеет координаты 5 ,0, 0». Действительно, выбрав иное начало отсчета (пулковский меридиан вместо гринвичского, сотворение мира вместо Рождества Христова, среднеевропейское время вместо московского, иную координатную систему, мы получили бы совсем иные числа: ноль градусов, 7195 год (а сам Ньютон получил бы 5675 год) и т. д.

Наоборот, в последующих утверждениях мы имеем дело с типичными скалярами: «Санкт-Петербург на 30 градусов восточнее Лондона», «Сегодня продолжительность дня в Москве составляет 12 часов», «Знаменитые «Начала» Ньютон опубликовал в возрасте 44 лет», «Расстояние между точками А и О равно 5».

Таким образом, всякому скаляру может быть поставлено в соответствие число, но не всякое число может являться скаляром. Другое дело, что при первоначальном изучении векторной алгебры на эти «тонкости» может быть и не следует заострять внимание учеников, но преподаватель это знать должен.

Об определении векторных величин. Вектором называют величину, характеризуемую числовым значением, направлением в пространстве и складывающейся с другой, себе подобной величиной геометрически.

Следует обязательно подчеркнуть, что последняя часть определения является не свойством вектора, что нередко утверждается, но именно неотъемлемой частью определения. Два первых требования необходимы, но недостаточны. В незнании этого заложен источник многочисленных ошибок.

Рассмотрим такую физическую величину как сила тока. Эта величина характеризуется, как известно, числом ( I = Δq/Δt) и направлением (от плюса к минусу). Однако известно, что токи в точке, где цепь разветвляется, складываются алгебраически, а не геометрически (первый закон Кирхгофа). Вот и получается, что сила тока, оказывающаяся вектором по «урезанному» определению, в действительности является скаляром. Поэтому следует избегать таких неточных, а, значит, и неправильных определений.

Вообще говоря, определения не могут быть правильными или неправильными, точными или неточными (запрещается только их внутренняя противоречивость). Кто, например, может запретить автору любой книги считать, что скаляр – это просто любое число, а вектор – это величина, характеризующаяся числом и направлением? Никто не может! Это – его право. Точно также автору никто не может запретить понимать под числом пи не 3,14…, а, например, 8,395 или измерять расстояние не в метрах, а в попугаях. Находясь в этой своей, если так можно сказать, «понятийной системе координат», автор может весьма комфортно себя чувствовать, делая одно открытие за другим. Но при общении с другими исследователями, находящимися в другой «понятийной системе», ему постоянно придется объяснять, что именно он понимает под тем или иным определением (термином, буквой…).

Даже при обозначении какой-то физической величины или действия над ней многие стараются использовать такие символы (буквы, знаки умножения, сложения, интегрирования и т.д.), которые «приняты на вооружение» большинством физиков. Так, общепринято обозначать заряд буквой q, массу – буквой m, и т. д. Даже в таких «мелочах», когда вместо общепринятого обозначения площади буквой S, некоторые авторы используют буквы A или F (а такое встречается в книгах по технической механике), многие испытывают психологический дискомфорт. А чего уж говорить, если речь идет об определении каких-то категорий, процессов… Тут и запутаться недолго. В общем, не такая уж это и безобидная вещь – пренебрежение общепринятыми соглашениями. Попробуйте, эксперимента ради, предложить ученикам измерять скорость в попугаях, деленных на секунду или под пи понимать число 8,395. Я догадываюсь, какова будет их реакция.

Во избежание ничем не оправданной путаницы, преподаватель должен жестко подходить к тому, что написано на «ярлыке», который мы наклеиваем на то или иное свойство изучаемого объекта. Все сказанное относится и к определению скалярных и векторных величин. Для них разработан соответствующий математический аппарат, им можно сопоставить конкретные физические свойства. Конечно, с течением времени некоторые понятия корректируются, уточняются, но всегда в рамках общих соглашений. Однако на сегодняшний день все определения, отличающиеся по содержанию от общепринятых, будут являться неточными и неверными.

Что касается обозначения векторных величин, то следует предостеречь учеников от такого их обозначения, когда над буквой вектора ставится черточка. И, хотя такой символ векторной величины давно ушел в прошлое, все-таки иногда его можно еще встретить. Черточкой над буквой принято обозначать средние значения величин, а для обозначения векторов применяется только стрелка над буквой (речь, конечно, идет о рукописном варианте, в печатном варианте буквы, обозначающие векторы имеют жирный шрифт).

Сложение векторов. Основным правилом сложения векторных величин является «правило многоугольника». На него и надо сделать акцент, а не на «правило параллелограмма». Причем следует подчеркнуть, что слагаемые векторы могут быть как угодно ориентированы в пространстве. При таком подходе теряет смысл раздельно излагать, например, плоскую и пространственную системы сил. Кроме того, следует внушить студентам, что изучаемые действия над векторами – это действия над абстрактными образами (грубо говоря, просто над стрелками). В векторной алгебре никакой вектор не наделяется никаким физическим содержанием. Но студенты также должны понимать, что эти абстрактные образы являются обобщением конкретных свойств многих физических объектов. Изучив действия над математическими образами, учащимся не составит никакого труда перенести эти действия на реальные свойства: скорости, силы, напряженности полей и т. д.

После такого подхода к изучению действий над векторами иногда приходится слышать от самих учеников такой, например, вопрос: «Зачем же тогда в учебниках каждый раз заново изучается операция сложения (вычитания, проектирования…) скоростей, сил, расположенных на плоскости, сил, расположенных в пространстве, сложение моментов и т. д.), если все они складываются по одним и тем же правилам, разработанным для «математических» векторов?». Вот и подумайте, как отвечать на подобные справедливые вопросы, а заодно и над тем, сколько часов в дальнейшем сэкономит такая изначальная «массированная векторная подготовка», и на сколько стали бы тоньше учебники, если их как следует «выжать».

При изучении именно сложения векторов следует особо подчеркнуть, что в основе изучаемой науки лежит эксперимент. Отнести ту или иную физическую величину к скалярам или векторам можно только на основе экспериментов (сложение электрических токов, о чем я уже говорил – тому пример). Распространенные суждения типа «Силы (ускорения, скорости и т. д.) складываются геометрически, так как это – векторы» принципиально ошибочны. Причина и следствие здесь поменялись местами, телега поставлена впереди лошади. Следует говорить: «Установлено опытом, что сила характеризуется числовым значением, направлением и складывается с другой силой по правилу параллелограмма. Следовательно, сила – вектор и, описывая силы, можно использовать разработанный для векторов математический аппарат».

Вычитание векторов. Здесь вначале надо дать определение обратного (противоположного) вектора, а затем просто заменить операцию вычитания операцией сложения. При таком подходе обычно у учащихся не возникает никаких вопросов. Но именно при изучении вычитания векторов самое время будет сказать ученикам о том, что не бывает ни положительных, ни отрицательных векторов. Знак же «минус» перед символическом обозначении вектора указывает только на то, что речь идет об обратном векторе, но вовсе не о том, что бывают отрицательные (а, значит, и положительные) векторы.

Векторы в отличие от скаляров бессмысленно также сравнивать между собой, используя понятия «больше» или «меньше», то есть записи: а < b или a > b лишены смысла (к модулям векторов это, конечно, не относится). Надо всегда помнить, что вектор – сложный объект, который одновременно характеризуется и величиной и направлением. Поэтому, если кто-то надумает сравнивать между собой векторы по признаку «больше – меньше», то тогда ему придется сравнивать по этому признаку не только модули векторов, но и их направления, а это, согласитесь, бессмысленно. А вот равными между собой векторы быть могут. Это – такие векторы, которые имеют одинаковые модули и одинаковые направления.

Умножение вектора на скаляр. В большинстве учебников дается следующее определение этой операции: «При умножении вектора а на скаляр k получается новый вектор kа, который имеет модуль в |k| раз больший модуля данного вектора. Направлен вектор kа в сторону вектора а, если k > 0 и в противоположную сторону, если k < 0». Очевидно, следует говорить, что модуль нового вектора не в | k | раз больше, а в | k | раз отличается от модуля исходного вектора. Думаю, что это уточнение пояснений не требует.

Скалярное умножение векторов. Эта операция встречается при введении понятия работы, потока вектора, при решении задач. Обычно трудностей в понимании этого действия не вызывает. Следует только обратить внимание учеников на обозначение этой операции (точка, но не косой крест, либо круглые скобки, в которые помещаются перемножаемые векторы). Полезно показать, что скалярное произведение вектора самого на себя даст квадрат его модуля (это пригодится, когда, например, будет рассматриваться вопрос о кинетической энергии).

Векторное произведение двух векторов. На эту операцию следует обратить особое внимание, во-первых, в силу ее важности при изучении некоторых понятий механики и электротехники, а, во-вторых, в силу некоторых затруднений ее усвоения учениками. Видимо, последнее соображение и заставляет авторов некоторых учебников закрывать глаза на ее применение, подчас, несмотря на то, что в начале курса эта операция иногда рассматривается. В результате некоторые важнейшие физические понятия извращаются до неузнаваемости. Судите сами.

Фундаментальное понятие момента силы относительно точки во всех школьных учебниках, а также практически во всех учебниках для техникумов и колледжей трактуется следующим образом (возможны несущественные отличия). «Моментом силы относительно точки называется взятое со знаком плюс или минус произведение модуля силы F на кратчайшее расстояние L от точки до линии действия силы, то есть М = ± FL».

Посмотрите на эту формулу и задумайтесь, что же это за величина такая момент силы: вектор это или скаляр? То, что автор такой записи не считает это понятие вектором сразу видно хотя бы по обозначению этой величины (над символом нет стрелки, и он не выделен жирным шрифтом). Символ ± тоже говорит не в пользу векторного характера момента силы (векторы не бывают ни положительными, ни отрицательными), да и произведение модуля силы на расстояние (сугубо скалярные величины) тем более не позволяет считать момент силы вектором.

Думаете, момент силы – это скаляр? Вынужден разочаровать: здесь положение ничем не лучше, чем с векторным статусом этого понятия: уж очень знак минус смущает. Действительно, откуда он мог взяться, если перемножаются сугубо положительные величины? Так что же это за величина-то такая? Это знают только авторы, предлагающие такое определение момента (да и то сомневаюсь в их толковом ответе). И все потому, что они просто не поверили в возможность ученика понять операцию векторного произведения двух векторов в применении, как к математическим образам, так и к физическим понятиям. Зато эти же авторы нисколько не стесняются проявить свои математические познания, предлагая, например, бедным студентам техникумов и колледжей рассчитывать перемещения сечений балок с помощью так «необходимого» студентам интеграла Мора, отечески «успокаивая» их тем, что это проще, чем решать двойные интегралы, формулы которых зачем-то приводятся. Кому нужна эта математическая бахрома, если вы не можете дать четкого и толкового определения, в общем-то, несложного, но чрезвычайно важного физического понятия?

Вернемся к моменту силы относительно точки. Конечно, это – вектор, и об этом все давным-давно знают. И равен этот вектор векторному произведению радиус-вектора точки приложения силы относительно полюса на вектор силы, то есть M = r x F. И направление этого вектора легко определяется по известному (даже ученикам) правилу буравчика. А вот, что касается плюса или минуса, то это уже зависит от направления оси, на которую я буду проектировать вектор M. И речь тут уже идет, как вы понимаете, не о моменте силы и не о его модуле, а о проекции этого вектора, и знаки эти зависят только от моей воли, а не от того, как уверяют авторы, поворачивает ли момент наше тело по или против хода часовой стрелки, поскольку направление оси, на которую проектируется вектор момента, я выбираю по своей прихоти.

Не забывайте также, что «намудрив» с понятием момента силы (хотели, как лучше…), авторам уже ничего не остается, как нести такую же ересь и в отношении момента пары сил. А ведь ученикам еще и задачи порешать не мешало бы для закрепления данного понятия. Как же их решать, если само понятие момента, данное этими авторами, вовсе этим понятием не является?

Вот и получается, что лучше уж совсем ничего не знать о моменте силы, чем иметь такие знания.

Не лучшим образом обстоит дело и с понятием угловой скорости, которая тоже «как на грех» является вектором. И связь между тремя векторами: вектором линейной скорости v, вектором угловой скорости ω и радиус-вектором r вращающейся точки тоже определяется правилом векторного произведения, а именно v = ω x r.

Кстати, многие авторы умалчивают о том, что угловая скорость – вектор тоже, видимо, по причине их беспокойства непонятливостью учеников. Ведь здесь же надо им как-то объяснить, почему сами углы векторами не являются, а вот их дифференциалы – почему-то векторы. Куда проще написать, что ω = φ/t, чем ω = dφ/dt. Но ведь всегда может найтись (и слава Богу!) какой-нибудь любознательный ученик, который возьмет да и спросит: «Линейная скорость – это вектор (Вы нам сами об этом говорили), а как насчет угловой скорости? Тоже ведь скорость как-никак…». И ведь бывает, спрашивают!

Ну, и, конечно, следует сказать учащимся, как обозначается векторное произведение: либо косым крестом, в отличие от точки, как в случае скалярного произведения, либо перемножаемые векторы помещаются в квадратные скобки. Одновременное применение и креста и квадратных скобок, то есть [axb] используется крайне редко.

Скалярная проекция вектора на ось. Необходимость изучения этой операции очевидна. Дело в том, что некоторые физические законы имеют векторный характер, а при решении тех или иных практических задач часто бывает необходимо получить конкретное число. Переход от векторов к их модулям без проектирования и нахождения проекций в большинстве случаев невозможен. Поэтому студенты должны не только знать, что такое проекция вектора, но и уметь ее находить.

Учащиеся также должны понимать, что, когда говорят «проекция вектора», имеют в виду скалярную проекцию. Слово «скалярная» в определении обычно опускается. И в то же время, эта проекция скаляром не является (хотя и называется скалярной проекцией), несмотря на утверждения многих авторов, что проекция вектора – это скаляр. Дело в том, что эта проекция зависит от выбора системы координат, и в разных системах проекции одного и того же вектора будут разными. Скаляр же – величина инвариантная по отношению к любым координатным системам.

О проектировании векторов говорится во всех учебниках, без этого просто невозможно изучение предмета. Другое дело, что большинство авторов идут по самому неоптимальному пути: проекции вводятся для конкретных физических образов (скоростей, сил и т. д.), вместо того, чтобы в самом начале курса рассмотреть вектор (и все, что с ним связано) как математический образ. Если этого не сделать, то студенты оказываются в положении, когда они «за деревьями не видят леса». Однако мы об этом уже говорили. Сейчас же я покажу, как некоторые авторы используют понятие проекции вектора на ось при изучении некоторых вопросов механики.

Приведу фрагмент изложения вопроса о растяжении и сжатии, взятый из одного из учебников по технической механике для техникумов. «…целесообразно договориться о правиле знаков для проекций внешних сил при определении нормальной силы в сечении: проекции внешних сил, направленных от сечения, положительны и, наоборот, проекции внешних сил, направленных к сечению, отрицательны». Авторы считают этот договор настолько важным, что сочли необходимым выделить курсивом некоторые слова и фразы своего предложения.

О чем, собственно, договор? А договор, по сути, о том, чтобы поставить жирный крест на понятии проекции вектора и его правильном использовании. Действительно, даже школьнику понятно, что бессмысленно говорить о проекции, не определив ось, на которую это проектирование выполняется. Авторы же этим грубо пренебрегают. Далее. Рассматриваемые авторами силы, направленные «от сечения», действуют в противоположных направлениях, поэтому они никак не могут иметь один и тот же знак своих проекций. То же самое касается и сил, направленных «к сечению».

Зачем выдумывать какое-то «правило знаков», когда можно (и нужно!) просто-напросто выбрать любую ось и спроектировать на нее все необходимые силы? Получается, что вместо стройного и последовательного изложения курса, перед нами предстает мозаичная картина несвязанных между собой «правил», ничего не дающих, кроме путаницы в головах учащихся.

Векторная проекция вектора на ось. Введение этого понятия связано с необходимостью знания студентами основной формулы векторной алгебры, которая, в свою очередь, используется как при решении задач, так и при изложении некоторых тем. Кроме того, векторная проекция используется и при разложении вектора на составляющие.

Проще всего ввести это понятие как произведение скалярной проекции вектора на орт соответствующей оси. Обозначение векторной проекции такое же, как и скалярной, но с «векторной атрибутикой». Например, если дан вектор а, то его скалярная проекция, например, на ось Х обозначается как ax, а векторная будет иметь обозначение ax (в рукописном варианте, как известно, вместо жирного написания буквы над ней ставится стрелка).

Следует приучать учеников к корректному использованию обозначений, связанных с векторными величинами. В частности, нужно обратить их внимание на традиционно существующую «чехарду» в обозначениях векторов, их составляющих и модулей, когда речь идет о нормальных и тангенциальных ускорениях. В литературе нередко и сам вектор нормального ускорения, и его проекция на нормаль, и модуль обозначаются одним и тем же символом аn. Аналогично и для касательного ускорения. Студенты должны знать, что:

aτ — это вектор касательного ускорения,

an — это вектор нормального ускорения.

aτ и an являются векторными проекциями полного ускорения а на касательную ось и ось нормали соответственно,

aτ — это проекция (скалярная!) касательного ускорения на касательную ось,

an — это проекция (скалярная!) нормального ускорения на ось нормали,

| aτ |- это модуль вектора касательного ускорения,

| an | — это модуль вектора нормального ускорения.

Нахождение модуля вектора по его проекциям. С точки зрения математики, тут все просто: модуль вектора равен корню квадратному из суммы квадратов его проекций. По сути – это обычная теорема Пифагора. Необходимость в применении этой формулы возникает всегда, когда в задаче нужно найти модуль какой-то векторной величины, например, силы. По ходу решения задачи находятся проекции, а потом и сам модуль. Здесь важно приучить студентов неукоснительно применять данную формулу, даже, если вектор имеет одну составляющую с положительной проекцией на выбранную ось.
Такая «дотошность» приучит студентов к математической и физической строгости, к пониманию того, что никаких «мелочей» и никакого «заметания мусора под ковер» в науке нет и быть не должно.

Все изложенное, конечно, не является чем-то новым для преподавателей тех дисциплин, где встречаются векторные величины. Математическую сторону поднятых вопросов знают преподаватели математики.

Целью же этой статьи является привлечение учителей к критическому анализу учебников и пособий с точки зрения поднятых вопросов и к пониманию необходимости единого подхода в применении методов векторного анализа. А это возможно при углублении межпредметных связей, на что должны обратить внимание преподаватели математики и физики конкретного учебного заведения.

Чернов Николай Сергеевич

Пожалуйста, не забудьте поделиться о прочитанном со своими друзьями в соц. сетях (см. кнопки ниже).

 

Учебно-методические карты по теме «Векторы» 1 курс СПО

УЧЕБНО-МЕТОДИЧЕСКАЯ КАРТА _63___

Математика

Тема занятия: Понятие вектора в пространстве. Сложение и вычитание векторов. Умножение вектора на число.

Вид занятия: Лекция. Время: 90_

Учебная: Формирование знаний о векторах в пространстве (равенство

векторов, коллинеарность, компланарность векторов)

Формирование умений складывать вектора, умножать вектор на число.

Применять изученный материал к решению задач.

Воспитательная: воспитание ответственности, коллективизма,

уважительного отношения к мнению однокурсников, умения выражать

и отстаивать собственное мнение.

Межпредметные

связи

Обеспечивающие: математика, планиметрия, черчение, тригонометрия.

Обеспечиваемые: алгебра и начала математического анализа,стереометрия,

физика.

Обеспечение занятия.

А. Наглядные пособия:каркасные модели стереометрических фигур.

Б. Раздаточный материал: учебники, справочный материал, карточки.

В. Технические средства обучения: ноутбук, проектор.

Г. Учебные места: лекционная аудитория № 116.

Д. Литература:

Основная: 1. Учебник «Геометрия 10-11» Л.С. Атанасян, В.Ф. Бутузов, С. Б.

Кадомцев, Л. С. Киселева, Э. Г. Позняк. М. «Просвещение»,2014

2. «Математика» М.И. Башмаков, М., 2014

Дополнительная:

3. «Тесты по геометрии к учебнику Л.С. Атанасяна» Ю.А.Глазков,

Л.И.Боженкова, М.,2012

4. «Контрольные работы по геометрии к учебнику Л.С.Атанасяна», Ю. П. Дудницын, В. Л. Кронгауз, М., 2009.

СОДЕРЖАНИЕ ЗАНЯТИЯ

элемента

Элементы занятия, учебные вопросы, формы и методы обучения

Добавления,

изменения,

замечания

1

2

3

I.

Организационная часть

1)

Приветствие. Проверка присутствующих.

2)

Оценка внешнего вида. Проверка готовности группы и аудитории к занятию.

3)

Инструкции по Т.Б.

II.

Актуализация опорных знаний студентов.

Метод: фронтальный опрос, беседа. Вопросы студентам.

1.

Что такое вектор?

2.

Что такое абсолютная величина вектора?

3.

Какие векторы называются одинаково направленными,

противоположно направленными?

4.

Какие векторы называются коллинеарными?

Условие коллинеарности.

III.

Мотивация учебной деятельности методом

объяснения. Метод: Сообщение, объяснение.

1)

Сообщение темы, целей, задач занятия.

2)

Практическая значимость темы.

IV

Изложение и первичное осмысление нового

материала: сообщение новых знаний методом

объяснения с элементами беседы.

План занятия:

1)

Определение вектора. С понятием вектора на плоскости мы уже сталкивались. Мы говорили, что есть такие величины, для которых важно не только численное значение, но и направление, например, сила, скорость и т. д. Такие величины мы называли векторными или просто векторами. В математике вектор изображается в виде направленного отрезка

Рис. 1. Вектор 

Вектором называется направленный отрезок, он имеет направление и величину.

Длина вектора соответствует длине отрезка, задающего этот вектор.

2)

Равные векторы. → Теперь введем второй вектор CD, обозначим его как вектор: b

Рис. 2. Коллинеарные векторы.

Если прямые AB  и CD  параллельны (или совпадают), то векторы a и b  коллинеарны.

Коллинеарные векторы могут быть противонаправлены: 

→ → → →

а ↑↓ b (рис. 2) или сонаправлены (а ↑↑ b ).

Определение. Равными называются коллинеарные сонаправленные векторы, длины (модули) которых равны.

→ → → →

Имеем вектор АВ = a  и вектор : А1В1= а

Рис. 3. Равные векторы

Заданные векторы равны, т. к. они коллинеарны, сонаправ

→ →

лены и их длины равны: │a.│=│a1│. Существует также нулевой вектор (0), т. е. вектор нулевой длины, он

изображается точкой.

Проводя аналогию с числами: мы знали число а и противоположное ему число – а, это были такие числа, сумма которых равна нулю: а + (-а). Аналогичное понятие существует и для векторов (рис. 4).

Рис. 4. Противоположные векторы

→ →

Задана точка О и два вектора: ОА и ОВ. Эти векторы имеют одинаковую длину, принадлежат одной прямой – коллинеарны – и противонаправлены. Такие векторы в сумме составляют нулевой вектор:

Кроме того: С физической точки зрения это можно представить следующим образом: если с равной силой тянуть предмет одновременно в две противоположные стороны, то он никуда не сдвинется.

4)

Сложение векторов.

Отметим, что сложение векторов производится аналогично планиметрии, только все действия выполняются в пространстве.

Итак, пусть заданы два произвольных вектора в пространстве (рис. 5):

Рис. 5. Произвольные векторы в пространстве

Определим, что же называется суммой двух этих векторов.

Точно так же, как в планиметрии, из любой удобной точки, назовем ее точкой А, можно единственным образом отложить вектор, равный вектору . Напомним, что заданные векторы, как и любые другие, свободны, важно лишь направление и длина, сам вектор можно параллельно переносить в любое место как на плоскости, так и в пространстве. Так, мы получили вектор  – в результате действия вектора  точка А переместилась в точку В. Теперь из точки В откладываем единственно возможным образом вектор , получаем вектор  – так, в результате действия вектора  точка В переместилась в точку С. В результате точка А переместилась в точку С, получен вектор , который и называется суммой векторов  и  (рис. 6).

Рис. 6. Сумма двух векторов в пространстве

Так, получено правило треугольника для сложения векторов в пространстве.

Правило треугольника

Из любой точки пространства (точка А) откладываем первый вектор, из конца первого вектора (точка В) откладываем второй вектор и получаем точку С. Вектор, соединяющий начало первого вектора (точка А) и конец второго (точка С), и будет результирующим.

5)

Вычитание векторов.

Определение :

Разностью двух векторов называется такой третий вектор, который, будучи сложенным со вторым вектором, даст первый вектор.

Введем разность векторов  и , для этого сложим вектор  с противоположным вектором :

Итак, из произвольной точки А откладываем вектор , получаем точку В. Чтобы получить вектор  мы строим вектор, равный вектору  по длине, но противонаправленный. Полученный вектор откладываем из точки В – получаем точку D. Вектор  и будет искомым вектором разности.

Проиллюстрируем (рис. 7):

Рис. 7. Вычитание двух векторов в пространстве

Построим на заданных векторах  и  параллелограмм (рис. 8):

Рис. 8. Параллелограмм на двух заданных векторах

Т. к. вектор ; аналогично .

По правилу треугольника:

Так, одна из диагоналей параллелограмма, построенного на двух векторах, соответствует сумме этих векторов.

Рассмотрим разность векторов. По правилу треугольника:

Так, вторая диагональ параллелограмма, построенного на двух векторах, соответствует разности этих векторов.

6)

Умножение вектора на число.

Необходимо построить вектор .

Видим, что перед некоторыми векторами стоят численные множители. Напомним, что при умножении вектора на число получаем сонаправленный вектор, длина которого – это длина исходного вектора, умноженная на заданное число. Получим векторы  и . Вектор  сонаправлен с вектором , длина его в три раза больше. Вектор  противонаправлен вектору , длина его в два раза больше. Проиллюстрируем (рис. 6):

Рис. 9. Умножение вектора на число

Приступаем к сложению. Из произвольной точки А откладываем полученный вектор  – получаем точку В. Из точки В откладываем вектор  – получаем точку С. Из точки С откладываем вектор  – получаем точку D. Согласно правилу многоугольника, вектор  соответствует искомому вектору :

Рис. 9. Сложение векторов по правилу многоугольника

V

Закрепление материала в ходе решения задач №320,

330, 344,.

VI

Определение результативности занятия в соответствии с

поставленной целью. Рефлексия.

VII

Определение задания для самостоятельной работы

обучающихся: № 322,347

Прочитать п.38- 42.Учебник «Геометрия 10-11».

Л.С. Атанасян, В.Ф. Бутузов, С. Б. Кадомцев,

Л. С. Киселева, Э. Г. Позняк,

(PDF) Введение в нелокальные пространства Дмитрий Малюгин, январь 2020, 1.0.1

1. Основные понятия

Нелокальная геометрия описывает метрическое аффинное пространство. Аффинность

пространства означает, что понятие элемента (пространства) является основным, а по-

нятие вектора — подчиненным. Наличие метрических свойств означает, что между объ-

ектами пространства определена операция скалярного произведения, а значит, и норма

объектов. Ненулевая норма элементов пространства характеризует нахождение элемен-

та за пределами пространства, задаваемого базисом. В этом и есть свойство нелокаль-

ности.

Выделенным объектом является нормаль, — она ортогональна всем векторам про-

странства. Норма нормали задает кривизну пространства. В плоском пространстве кри-

визна равна нулю и нормаль является нуль-вектором пространства. По аналогии с

определением проективной геометрии как ”евклидовой геометрии, пополненной поня-

тием бесконечно удаленной точки” [Прас], можно сказать, что нелокальная геометрия —

это аффинная метрическая геометрия, пополненная понятием нормали [Кост].

В работе можно выделить три основных части. В первой (разделы 1, 2) приведе-

ны основные геометрические свойства элементов пространств. Во второй (разделы 3,

4) рассматриваются базисы и их свойства. Третья часть (разделы 5, 6, 7) посвящена

особенностям линейных преобразований. Частные вопросы вынесены в приложение.

1.1. Преамбула

Отметим на листе бумаги (представляющего в данном случае двумерное евклидово про-

странство R2) несколько точек и построим матрицу квадратов расстояний qij =qji =

|xi−xj|2между всеми точками:

Q=

0q12 q13 …

q21 0q23 …

q31 q32 0…

… … … 0

(1.1.1)

В основе нелокальной геометрии лежит следующее наблюдение [Gow85]. Если коли-

чество точек меньше или равно четырем, то детерминант такой матрицы (при условии,

что точки не принадлежат одной окружности) будет отличен от нуля. А если количе-

ство точек равно или более пяти — то дистанционная матрица будет вырождена и ее

детерминант всегда (для любой конфигурации точек) будет равен нулю.

Матрица квадратов дистанций (квадрансов) Qмежду точками может быть выраже-

на через матрицу скалярных произведений (при условии коммутативности последнего):

Q=−2G. Невырожденная матрица скалярных произведений между элементами означа-

ет независимость элементов набора. Таким образом на плоскости независимыми являют-

ся четыре точки, а пятая может быть выражена как их линейная комбинация. Отсюда

делаем вывод, что в двумерном пространстве для полной характеристики элемента тре-

буется в общем случае четыре координаты.2
ight)который называется скалярным квадратом вектора


Для евклидовых пространств это означает, что квадратичная форма является положительно-определенной. евклидовые пространства рассматриваются как псевдо-евклидовые пространства – см. Обратите внимание, что если и тогда это нулевой вектор. В псевдоевклидовом пространстве, в отличие от Евклидового пространства, существуют векторы с отрицательным скалярным квадратом.
Как и в случае с термином евклидовое пространство, термин псевдоевклидовое пространство может быть использован для обозначения аффинного пространства или векторного пространства в зависимости от автора, причем последнее альтернативно называется псевдоевклидовым векторным пространством (см. различение “точка-вектор”).

Геометрия

Геометрия псевдоевклидового пространства последовательна, несмотря на некоторые неприменимые свойства евклидового пространства, в частности, это не метрическое пространство, как объясняется ниже. Аффинная структура остается неизменной, а значит, и понятие “линия”, “плоскость” и, как правило, аффинное подпространство (плоское), а также линейные отрезки.

Положительный, нулевой и отрицательный скалярные квадраты

Нулевой вектор – это вектор, для которого квадратичная форма равна нулю. В отличие от евклидового пространства, такой вектор может быть ненулевым, в этом случае это Я.
Если квадратичная форма неопределенна, то псевдоевклидовое пространство имеет линейный конус нулевых векторов, заданных x – q(x) 0. Когда псевдоевклидовое пространство предоставляет модель для пространства-времени (см. ), нулевой конус называется светлым конусом начала отсчета.
Нулевой конус разделяет два открытых множества, для которых предполагается стандартная топология, соответственно, для которых и If – набор векторов, для которых подключен. Если же оно состоит из двух разобщенных частей, то для векторов, для которых, если оно заменено на

Интервал

Квадратичная форма соответствует квадрату вектора в евклидовом случае. Чтобы определить норму вектора (и расстояние) инвариантным образом, нужно получить квадратные корни скалярных квадратов, что приводит к тому, что на возможные воображаемые расстояния видны квадратные корни отрицательных чисел. Но даже для треугольника с положительными скалярными квадратами всех трёх сторон (квадратные корни которого являются вещественными и положительными), неравенства треугольника не существует вообще.
Таким образом, в псевдоевклидовой геометрии избегаются термины “норма” и “расстояние”, которые могут быть заменены скалярным квадратом и интервалом соответственно.
Хотя для кривой, тангенциальные векторы которой имеют скалярные квадраты одного знака, определяется длина дуги. В ней есть важные приложения – например, видим соответствующее время.

Вращения и сферы

Вращение (математика). Такие “повороты” сохраняют форму и, следовательно, скалярный квадрат каждого вектора, включая, является ли он положительным, нулевым или отрицательным.

Симметричная билинейная форма

Квадратичная форма порождает симметричную билинейную форму, определяемую следующим образом –

Квадратичная форма может быть выражена в виде билинейной формы – х, х.
Когда x, y 0, то и являются ортогональными векторами псевдоевклидового пространства.
Эту билинейную форму часто называют скалярным продуктом, а иногда “внутренним продуктом” или “точечным продуктом”, но она не определяет пространство внутреннего продукта и не обладает свойствами точечного продукта евклидовых векторов.
Если и ортогональны, а затем гиперболически-ортогональны к
Стандартная основа реального пространства ортогональна. В псевдоевклидовом пространстве, для которого билинейная форма неопределена, ортонормальных оснований нет, поскольку их нельзя использовать для определения векторной нормы.

Пространства и ортогональность

Для (позитивно-размерного) подпространстваА предполагается линейное подпространство, но те же выводы справедливы и для аффинной плоскости с единственным осложнением – квадратичная форма всегда определяется по векторам, а не по точкам. псевдоевклидового пространства, когда квадратичная форма ограничена следующими тремя случаями-
U либо положительный, либо отрицательный определенный. Тогда, по сути, это евклидово (до знака ).
U неопределённо, но не дегенеративно. Тогда, сам по себе псевдо-евклидовый. Это возможно только если, что означает, чем является плоскость, то это называется гиперболической плоскостью.
U – дегенерат.
Одним из наиболее жгучих свойств (для евклидовой интуиции) псевдоевклидовых векторов и квартир является их ортогональность. Когда два ненулевых евклидовых вектора ортогональны, они не являются коллинеарными. Пересечения любого евклидового линейного подпространства с его ортогональным дополнением являются подпространством пространства нулевого вектора. Но определение из предыдущего подраздела сразу же подразумевает, что любой вектор нулевого скалярного квадрата ортогонален сам по себе. Следовательно, изотропная линия ν, порожденная нулевым вектором ν, является подмножеством его ортогональной комплементации
Формальное определение ортогональной комплементарности векторного подпространства в псевдоевклидовом пространстве дает совершенно четко определенный результат, удовлетворяющий равенству за счет недегенеративности квадратичной формы. Это просто условие
– или, эквивалентно, все пространство,
который может быть разорван, если подпространство содержит нулевое направление. На самом деле, он не является нулевым только в том случае, если квадратичная форма ограничена дегенерацией. В то время как подпространства образуют решетку, как и в любом векторном пространстве, эта операция не является ортокомплексом, в отличие от внутренних пространств произведения.
Для подпространства, состоящего целиком из нулевых векторов (что означает, что скалярный квадрат, ограниченный равным ), всегда держит-
– или, эквивалентно,
Такое подпространство может иметь до размеров. Томас Э. Сесил (1992) Геометрия сферы лжи, стр. 24, Университетский Шпрингер.
Для (положительного) евклидового – подпространства его ортогональное дополнение является -мерным отрицательным “евклидовым” подпространством, и наоборот.
Вообще, для -мерного подпространства, состоящего из положительных и отрицательных измерений (см. закон инерции Сильвестра для уточнения), его ортогональное “дополнение” имеет положительные и отрицательные измерения,⊥) d0, что само по себе является прямым следствием определения U⊥ и закона Сильвестра, но было бы лучше найти источник. — в то время как остальные дегенерируют и образуют пересечение.

Параллелограммное право и пифагорейская теорема

Закон о параллелограмме принимает форму
– q(x) + q(y) frac(q(x + y) + q(x – y)).
Используя квадрат суммы тождества, для произвольного треугольника можно выразить скалярный квадрат третьей стороны из скалярных квадратов двух сторон и их билинейную форму изделия-
– q(x + y) q(x) + q(y) + 2langle x, y
angle.
Это показывает, что для ортогональных векторов псевдо-евклидовый аналог пифагорейской теоремы имеет…

Angle

Как правило, абсолютное значение x, y билинейной формы на двух векторах может быть больше, чем равно, или меньше. Это вызывает аналогичные проблемы с определением угла (см. ), как и для расстояний.
Если (только один положительный член в ), то для векторов положительного скалярного квадрата –

что позволяет определить гиперболический угол, аналог угла между этими векторами через обратный гиперболический косинус.2,
Геометрия, связанная с этой псевдометрикой, была исследована Пуанкаре.Г. Пуанкаре (1906 г.) “О динамике электрона, Рендиконти дель Циркол Математико ПалермоВ”. A. Розенфельд (1988 г.) “История неевклидовой геометрии”, стр. 266, Исследования в истории математики и физики 12, Шпрингер Его ротационная группа – это группа Лоренца. Группа Пуанкаре также включает в себя переводы и играет ту же роль, что и евклидовые группы обычных евклидовых пространств.
Другим псевдоевклидовым пространством является плоскость, состоящая из сплит-комплексных чисел, снабженная квадратичной формой

Это самый простой случай неопределенного псевдоевклидового пространства и единственный случай, когда нулевой конус рассекает пространство на четыре открытых множества. Группа состоит из так называемых гиперболических вращений.

Текст этой страницы составлен на основе перевода страницы из Wikipedia Pseudo-Euclidean space

Материал использован на основании Creative Commons Attribution-Share-Alike License

Векторы — презентация на Slide-Share.ru 🎓

1

Первый слайд презентации: Векторы

Семинар 2

Изображение слайда

2

Слайд 2

Векторами называются математические объекты ( a, b, c, …), для которых определено выполнение двух алгебраических операций: — сложение двух векторов a + b = c — умножение вектора на число  • а = b вектор того же типа Множество векторов одного типа называется векторным пространством условия линейности  ( a + b ) =  a +  b ( +  ) a =  a +  b линейное пространство (ЛП) или линейное векторное пространство (ЛВП) Если ЛВП может, наряду с группами симметрии, служить еще одним примером математических структур, представляющих собой замкнутые множества однотипных и упорядоченных определенным образом (с помощью алгебраических операций) объектов

Изображение слайда

3

Слайд 3: Линейные комбинации

 a +  b +  c +….. = x Линейная комбинация (ЛК) векторов a, b, c,… c коэффициентами , , ,… всякая ЛК любых векторов некоторого ЛП также является вектором того же самого ЛП; любой вектор некоторого ЛП может быть представлен в виде ЛК нескольких векторов того же самого ЛП; в любом ЛП существует такой выделенный набор векторов, называемый базисным набором (или просто базисом ), что все, без исключения, векторы этого ЛП могут быть представлены как линейные комбинации этих выделенных базисных векторов. На векторы, выбираемые в качестве базисных, накладывается одно важное условие: они должны быть линейно независимы между собой (не должны выражаться друг через друга, т.е.: x ≠   y ). Выбор базиса неоднозначен и в любом ЛП возможных базисов бесконечно много, однако все они содержат одно и то же число базисных векторов. Это фундаментальное число называется размерностью ЛП.

Изображение слайда

4

Слайд 4: Координатное представление векторов

Координаты вектора — это числа х 1, х 2, х 3,…, являющиеся коэффициентами ЛК: х = х 1 е 1 + х 2 е 2 + х 3 е 3 +…., где е 1, е 2, е 3,… — базисные векторы (базис). контравариантный или контра -вектор ковариантный или ко -вектор С математической точки зрения контра — и ковекторы полностью эквивалентны, однако они могут использоваться для обозначения различных типов физических объектов.

Изображение слайда

5

Слайд 5: Операции над векторами

Умножение вектора на число Сложение двух векторов Линейная комбинация Координатное представление базисных векторов:

Изображение слайда

6

Слайд 6: Скалярное умножение векторов

 = ( x, y ) = x  y =  x | y  Результатом этой операции является число, называемое скалярным произведением векторов, значение которого можно рассчитать через координаты векторов х и у по формуле : Скалярный квадрат вектора : Вектор-строка – первый вектор. Вектор-столбец – второй!

Изображение слайда

7

Слайд 7: Метрические понятия в модели ЛВП

Скалярный квадрат вектора = квадрат модуля вектора модуль вектора = длина вектора Угол между двумя векторами (  ) : Метрические понятия в модели ЛВП При умножении вектора на некоторое число  его модуль (длина) увеличивается в  раз, тогда как ориентация (углы относительно всех остальных векторов) остается неизменной. Луч – подмножество векторов ЛП с одним и тем же направлением В каждом луче можно выделить один особый вектор, длина которого равна единице. Такой вектор служит представителем данного луча и называется нормированным.

Изображение слайда

8

Слайд 8: Нормирование вектора

При нормировке каждая координата вектора делится на величину модуля этого вектора. Например, трехмерный вектор х = (3, 5, 8) имеет длину: Для нормировки разделим все координаты на 10 и получим нормированный вектор х’ = (3/10, 5/10, 8/10 ) совпадающий по направлению с исходным (т.е. принадлежащий тому же лучу), но уже имеющий единичную длину.

Изображение слайда

9

Слайд 9: Комплексные векторы

В квантовой механике широко применяют векторы, координаты которых могут быть комплексными числами. В квантовой механике Ковекторы = бра — векторы. Обозначаются  x | Контравекторы = кет — векторы. Обозначаются | x  (от англ. — bracket — скобка ) Координаты бра- и кет-векторов (с одними и теми же номерами) являются комплексно сопряженными между собой. Два вектора, отличающиеся типом (бра- и кет-), и координаты которых взаимно комплексно сопряжены, называются сопряженными векторами (или, более полно, эрмитово сопряженными ), что отмечается верхним индексом ( + ).

Изображение слайда

10

Слайд 10: Скалярное произведение комплексных векторов

Особенность взаимно сопряженных комплексных чисел состоит в том, что их произведение, называемое квадратом модуля комплексного числа, всегда является действительным числом. Например: | Z | 2 = Z  Z * = (2 + 3i)(2 – 3i) = 2 2 + 3 2 = 13 Если перемножить (в смысле скалярного умножения) два вектора, координаты которых взаимно сопряжены, то квадрат модуля любого вектора будет не только действительным, но и положительным числом. Следовательно, из него всегда можно извлечь корень и определить длину (модуль) вектора. Если векторы-сомножители различны, то их скалярное произведение не будет действительным числом. Такие комплексные числа, являющиеся скалярным произведением двух комплексных векторов: С =  х | у  называются квантовомеханическими амплитудами и занимают центральное место в математическом аппарате квантовой механики.

Изображение слайда

11

Слайд 11: Функциональное представление векторов

Иногда приходится иметь дело с векторами многомерных ЛВП, имеющими очень много координат. Встречаются даже бесконечномерные пространства, как например, в квантовой механике. Особенность: соседние (в отношении их номеров) координаты имеют близкие значения. координаты вектора – значения некоторой непрерывной алгебраической функции. Функция – бесконечномерный вектор или функциональное представление вектора. волновые функции – функциональные представления квантовомеханических векторов состояния. Каждое значение волновой функции является некоторой координатой вектора состояния.

Изображение слайда

12

Последний слайд презентации: Векторы: Заключение

Векторы используются как математические модели некоторых физических или химических объектов. Одни и те же правила обращения с векторами мы можем применять для самых разных случаев. Универсальность векторной модели За абстрактными векторами могут стоять различные по природе объекты: геометрические векторы-стрелки, механические движения, химические реакции, смеси химических веществ и т.д. за абстрактными операциями сложения векторов и умножения их на число может стоять: сложение векторов-стрелок по правилу параллелограмма, перемещение объекта во времени или в пространстве, последовательное проведение химических реакций, смешивание нескольких веществ и т.д.

Изображение слайда

Математика — Векторы — Мартин Бейкер

Как и многие математические концепции, векторы можно понимать и исследовать по-разному.

Есть как минимум два способа смотреть на векторы:

  • Алгебраический — обрабатывает вектор как набор скалярных значений как единое целое с добавлением, вычитанием и скалярным умножением, которые работают со всем вектором.
  • Геометрический — вектор представляет величину, имеющую как величину, так и направление.

Мы можем абстрагироваться от различий в этих подходах и просто посмотреть, что всегда верно для векторов, когда мы это делаем, мы получаем набор аксиом, обычно в форме уравнений.Примером аксиомы для векторов является «закон распределенности»:

c (v 1 + v 2 ) = c v 1 + c v 2

, где v 1 и v 2 — векторы, а c — скаляр.

Эта аксиома важна, потому что она описывает линейное свойство векторов

Геометрические свойства

Вектор — это величина, имеющая как величину, так и направление, есть две операции, определенные с векторами, и обе имеют очень прямую геометрическую интерпретацию.Мы рисуем вектор как линию со стрелкой, а пока я назову конец без стрелки «началом» вектора, а конец со стрелкой — «концом» вектора.

  • Сложение векторов: чтобы сложить два вектора, мы берем начало второго вектора и перемещаем его в конец первого вектора. Сложение этих двух векторов представляет собой вектор от начала первого вектора до конца второго вектора.
  • Скалярное умножение изменяет длину вектора без изменения его направления.То есть мы «масштабируем» его на коэффициент умножения. Итак, скалярное умножение включает в себя умножение скаляра (одного числа) на вектор, чтобы получить другое число.

Мы можем рассматривать эти две операции: сложение векторов и скалярное умножение как определение линейного пространства (см. Евклидово пространство).

Итак, как вообще получить векторы? Мы могли бы принять уже существующую систему координат и определить все наши векторы в этой системе координат, или мы могли бы начать с набора базисных векторов и представить векторы как линейную комбинацию этих базисных векторов, то есть путем скалярного умножения и сложения базисные векторы мы можем создать любой вектор в пространстве при условии, что:

  • Базовых векторов столько, сколько измерений в пространстве.
  • Все базисные векторы независимы (не более двух в любой данной плоскости).

Таким образом, любая точка может быть идентифицирована по:

α В а + β В б

где:

  • α, β = скалярные множители
  • V a , V b = базисные векторы.

Итак, два скалярных множителя (α, β) могут представлять положение точки в терминах наших базисных векторов.Это приводит к способу работы с векторами чисто алгебраическим способом.

Алгебраические свойства

Алгебраический подход и его операции объяснены на этой странице, поэтому здесь мы просто дадим обзор.

Мы можем думать о векторе как о концепции массива на компьютерном языке, например,

  • Векторы имеют размер, равный количеству элементов в массиве.
  • Все элементы в векторе должны быть одного типа.
Вектор может отображаться в виде одного столбца
или рядом

Однако есть отличие от компьютерного массива, потому что в случае компьютера элементами массива могут быть любые допустимые объекты, при условии, что они все одного типа. В случае векторов элементы должны иметь определенные математические свойства, в частности, для них должны быть определены операции сложения и умножения с определенными свойствами.Свойства, необходимые для элементов вектора, заключаются в том, что они должны образовывать математическую структуру, известную как поле (см. Рамку справа). В математической терминологии это известно как вектор над полем, другими словами, вектор, элементами которого являются поля.

работа обозначение объяснение
дополнение В (а + б) = В (а) + В (б): сложение двух векторов выполняется путем сложения соответствующих элементов двух векторов.
скалярное умножение В (с * а) = с * В (а) скалярное произведение вектора получается путем умножения скалярного произведения на каждый из его членов по отдельности.

Эти операции взаимодействуют согласно свойству распределенности:

с * (b + c) = s * b + s * c

Что придает векторам линейное свойство. Теперь мы можем составить набор аксиом для векторов:

аксиома дополнение скалярное умножение
ассоциативность (а + б) + с = а + (б + в) (s1 s2) a = s1 (s2 a)
коммутативность а + Ь = Ь + а
распределительное устройство с * (b + c) = s * b + s * c
(s1 + s2) * a = s1 * a + s2 * a
идентификационный номер а + 0 = а
0 + а = а
1 а = а
обратное а + (- а) = 0
(-а) + а = 0

Где:

  • a, b, c — векторы
  • 0 — тождественный вектор
  • s, s1, s2 — скаляры
  • 1 — скаляр идентичности

Сравните это с аксиомами поля (на этой странице)

Векторы также могут иметь дополнительную структуру, определенную в терминах других определенных для них умножений, таких как скалярные произведения и перекрестные произведения, как мы увидим позже.Это необязательные операции, единственными обязательными операциями являются сложение и скалярное умножение.

Векторное обозначение

До сих пор мы показали вектор как набор значений в сетке, так как это более удобно на веб-странице html, но обычное обозначение вектора — это помещать значения в квадратные скобки:

Где:

  • x = компонент в измерении x.
  • y = составляющая в измерении y.
  • z = компонент по оси z.

Иногда, когда мы представляем весь вектор как символ, мы можем поместить стрелку над символом (в данном случае v ), чтобы подчеркнуть, что это вектор.

Или альтернативно, мы можем использовать следующие обозначения:

= a 1 x + a 2 y + a 3 z

Где:

  • x = единичный вектор в измерении x.
  • y = единичный вектор в измерении y.
  • z = единичный вектор в измерении z.

Первая форма более удобна при работе с матрицами, а вторая форму легче написать в текстовой форме.

Здесь «x», «y» и «z» — операторы, они часто могут использоваться в уравнениях аналогично переменным, но у них могут быть разные законы (например, умножение может не коммутировать). Это может быть удобным способом кодирования законов комбинирования векторов в обычной алгебре поиска.

Связь с другими математическими величинами

Мы можем расширить концепцию векторов (обычно добавляя дополнительные типы умножения для добавления к встроенному сложению и скалярному умножению), чтобы сформировать более сложные математические структуры, в качестве альтернативы мы можем рассматривать векторы как подмножества этих структур, например :

  • Как подмножество матрицы или тензора (матрица 1 на n или n на 1).Матрица — это двумерный массив с скалярным произведением.
  • Как подмножество мультивекторов (алгебра Клиффорда). Например, комплексные числа — это двухэлементные векторы с добавлением определенного типа умножения.

Чего мы не можем сделать, так это иметь вектор, элементы которого сами являются векторами. Это связано с тем, что элементы вектора должны быть математической структурой, известной как «поле», а вектор сам по себе не является полем, потому что он не обязательно имеет коммутативное умножение и другие свойства, необходимые для поля.

Тем не менее, было бы неплохо, если бы мы могли построить матрицу из вектора (нарисованного в виде столбца), элементы которого сами являются векторами (нарисованными в виде строки):

Чтобы создать матрицу путем объединения векторных структур, нам нужно сделать две вещи с «внутренним вектором»:

  • Нам нужно сделать транспонирование так, чтобы это была строка, а не столбец.
  • Нам нужна операция умножения, которая сделает его полем.

Для этого мы создаем «двойник» вектора, он называется ковектором, как описано на этой странице.

Векторы могут быть умножены на скаляры, даже если они являются отдельными объектами, векторы и скаляры не могут быть добавлены, например (только когда мы дойдем до алгебры Клиффорда), но мы можем определить тип умножения, называемый скалярным умножением, обычно обозначаемый ‘* ‘или скаляр может быть записан рядом с вектором с подразумеваемым умножением. Этот тип умножения требует одного вектора и одного скаляра. Скалярное умножение умножает величину вектора, но не меняет его направления, поэтому:

если есть,

vOut = 2 * vIn

где:

тогда vOut будет вдвое больше vIn, но в том же направлении.

Квадратичная структура на линейном пространстве

Однако этих линейных свойств самих по себе недостаточно, чтобы определить свойства евклидова пространства, используя только алгебру. Чтобы иметь возможность определять такие понятия, как расстояние и угол, мы должны определить квадратичную структуру.

Например, пифагор:

r 2 = x 2 + y 2 + z 2

в алгебраических терминах,

, если a — трехмерный вектор с основаниями e 1 , e 2 , e 3

a = a 1 e 1 + a 2 e 2 + a 3 e 3

так,

a • a = a 1 2 + a 2 2 + a 3 2

Другие векторные алгебры

В уже обсуждаемой векторной алгебре квадрат вектора всегда является положительным числом:

а х а = 0
a • a = положительное скалярное число

Однако мы могли бы определить одинаково действительную и непротиворечивую векторную алгебру, которая возводит в квадрат отрицательное число:

а × а = 0
a • a = отрицательное скалярное число

Мы также могли бы определить алгебру, в которой мы смешиваем измерения: квадрат с положительным, другой квадрат с отрицательным.Примером этого является квадрат Эйнштейна пространства-времени, пространства и времени в квадрате с разными значениями, если пространство квадратов в положительное значение, то квадрат времени в отрицательное и наоборот.

Приложения векторов

Для 3D программирования (тематика этого сайта) нас больше всего интересует с векторами из 2 или 3 чисел.

Вектор размерности 3 может представлять физическую величину, которая является направленной. например положение, скорость, ускорение, сила, импульс, и т.п.

Например, если вектор представляет точку в пространстве, эти 3 числа представляют положение в координатах x, y и z (см. системы координат). Где x, y и z — взаимно перпендикулярные оси в какое-то согласованное направление и единицы.

Трехмерный вектор может также представлять смещение в пространстве, например перевод в каком-то направлении. В случае библиотеки Java Vecmath эти это два класса: Point3f и Vector3f, оба производные от Tuple3f.(Обратите внимание на эти использовать числа с плавающей запятой, существуют также классы, оканчивающиеся на d, которые содержат двойные значения). Класс Point3f используется для представления абсолютных точек и Класс Vector3f представляет смещение. В большинстве случаев поведение этих классы одинаковы, насколько я знаю разница между этими классами когда они преобразуются матрицей Point3f будет транслироваться матрицей но Vector3f не будет.

Здесь мы разрабатываем следующие классы для хранения вектора и инкапсуляции операции, описанные здесь,

Можно было бы построить векторный класс, который мог бы содержать вектор любого измерение, но класс переменного измерения был бы менее эффективным.Поскольку мы когда речь идет об объектах в трехмерном пространстве, важнее работать с 2D и 3D векторы эффективно.

Другие векторные величины

Альтернативная интерпретация векторов

До сих пор мы думали о векторе как о позиции на 2,3- или n-мерном пространстве. сетка. Однако для некоторых физических ситуаций может не быть готово определенного Декартова система координат. Альтернативой может быть представление вектора как линейная комбинация из 3-х базисов:

σ 1
σ 2
σ 3

Эти основания не обязательно должны быть взаимно перпендикулярными (хотя в большинстве случаев они, вероятно, будут), но они должны быть независимыми друг от друга, другими словами, они не должны быть параллельны друг другу, и все 3 не должны быть в одной плоскости.

Итак, вектор в 3 измерениях может быть представлен как [a, b, c], где a, b и c представляют масштабирование 3-х базисов, чтобы сделать вектор следующим образом:

а σ 1 + б σ 2 + с σ 3

Обратите внимание, что если этот вектор представляет положение, то это будет относительное положение, т.е. относительно некоторой другой точки, если мы хотим определить абсолютную точку, мы еще нужно определить происхождение.

Итак, остается проблема, как определить основу, могут быть некоторые естественные определение их в проблемной области.В качестве альтернативы мы могли бы определить базируются как трехмерные векторы с использованием системы координат. Но зачем беспокоиться сделайте это, если у нас есть система координат, почему бы просто не представить векторы в это система координат? Что ж, мы могли бы захотеть изменить систему координат или перевести все векторы каким-либо образом (см. здесь). Например, мы могли бы захотеть представить точки на твердом объекте в некотором локальном системе координат, но твердый объект может сам двигаться относительно некоторого абсолютная система координат.

Дополнительная литература

Векторы могут управляться с помощью матриц, для пример переведен, повернут, масштабирован, отражен.

Есть математические объекты, известные мультивекторами, их можно использовать для выполнения многих задач, которые выполняют векторы, но у них нет некоторые ограничения (например, векторное векторное произведение ограничено 3-мя измерениями и не имеет обратного).

Существует также более общее семейство алгебр (меньше ограничений), чем «векторные пространства над полем», это «модули над кольцом».

Видеоурок: Скалярное умножение и единичные векторы

Стенограмма видео

В этом видео мы узнаем, как умножить вектор на скаляр и как найти единичный вектор в направлении любого заданный вектор путем деления вектора на скаляр. Мы рассмотрим оба из них в как в двух, так и в трех измерениях.

Давайте начнем с того, что вспомним, что мы означает скалярное умножение.Умножение вектора на скаляр или вещественное число называется скалярным умножением. Чтобы выполнить скалярное умножение, вам нужно умножить скаляр на каждый компонент вектора. Рассмотрим вектор 𝐕 с компоненты 𝑎, 𝑏 и 𝑐. А теперь представим, что мы хотим умножьте этот вектор на константу или скаляр. Делаем это так же, как если бы мы раздавали круглые скобки или раскрывали скобки. Вектор имеет компоненты 𝑘𝑎, 𝑘𝑏 и 𝑘𝑐.

При умножении вектора на скаляр, результат тоже вектор. Мы знаем, что любой вектор имеет и величина и направление. Умножение вектора на положительное число, отличное от единицы, изменяет величину, но не меняет направление. Умножение вектора на отрицательное один меняет направление на противоположное, но не меняет величину. Умножение на любой другой отрицательный число меняет направление и меняет величину вектора.

Теперь рассмотрим пару примеры, где нам нужно умножить вектор на скаляр.

Учитывая, что вектор 𝐀 равен отрицательный один, отрицательный восемь, найдите три 𝐀.

Умножение вектора на любое действительное число, как в этом случае, известно как скалярное умножение. Чтобы выполнить скалярную умножение, мы умножаем каждую составляющую вектора на скаляр. В этом вопросе нам нужно умножьте вектор отрицательную единицу, отрицательную восьмерку на три.Умножение положительного числа на отрицательное число дает отрицательный ответ. Следовательно, три умноженные на отрицательный — отрицательный три. Три умножить на восемь отрицательных равно отрицательному 24. Тройка, умноженная на вектор 𝐀 или вектор три 𝐀 равен отрицательным трем, отрицательным 24.

В следующем вопросе мы будем рассмотрим, что происходит, когда мы графически умножаем вектор на скаляр.

Вектор 𝐀 представлен следующий график.Какой из следующих графиков представляет отрицательные два 𝐀?

Вектор 𝐀 идет от начала до точка один, один. Это означает, что у него есть 𝑥-составляющая одного и 𝑦-составляющая одного. Вектор 𝐀 равен единице. Мы хотим умножить вектор 𝐀 на отрицательные два. Напомним, что при умножении вектор на скаляр, нам нужно умножить каждый из отдельных компонентов на скаляр. Умножение отрицательных двух на один дает нам два отрицательных.Следовательно, вектор, соответствует отрицательным двум 𝐀 отрицательным двум, отрицательным двум.

Давайте теперь рассмотрим пять вариантов нам даны и какие векторы они представляют. График (A) идет от начала координат к два отрицательных, два отрицательных. Это означает, что он соответствует вектор отрицательный два, отрицательный два. Это то же самое, что и отрицательные два 𝐀, что говорит о том, что это правильный график.

График (B) показывает вектор, отрицательные два.График (C) показывает вектор, два. График (D) показывает вектор, 0,5. График (E) показывает вектор 0,5, 0,5. Это подтверждает, что график (A) действительно представляют отрицательные два 𝐀. Это приводит нас к ключевому правилу, когда умножение на отрицательные скаляры. Когда мы умножаем любой вектор на отрицательный скаляр, отличный от отрицательного, вектор изменит направление и величина. Умножение вектора на отрицательное два, как в этом случае, удвоят величину, а направление вектора будет противоположным исходному направлению.Это можно показать на координате самолет по зеленой стрелке.

Остальные вопросы в это видео будет иметь дело с единичными векторами. Начнем с определения того, что мы означают единичным вектором.

Единичный вектор — это вектор, имеющий величиной один. Напомним, что трехмерный вектор с компонентами 𝑎, 𝑏, 𝑐 имеет величину, равную квадратному корню из 𝑎 в квадрате плюс в квадрате плюс в квадрате.Полоски абсолютных значений обозначают величина вектора. Чтобы найти единичный вектор с тем же направлением, что и данный вектор, мы делим на величину данного вектор. Единичный вектор записывается 𝐕 шапка. Это равно единице по величина 𝐕, умноженная на вектор 𝐕.

Рассмотрим вектор 𝐕 с Компоненты четыре, три. Величина вектора равна к квадратному корню из четырех в квадрате плюс три в квадрате.Четыре в квадрате равно 16, а три в квадрате равно девяти. Это означает, что величина вектор 𝐕 равен квадратному корню из 25. Обычно мы оставляем это в радикальная или грубая форма. Но поскольку квадратный корень из 25 является целое число, мы можем упростить так, чтобы величина вектора была равна пяти.

Таким образом, единичный вектор равен равно одной пятой, умноженной на вектор четыре, три. Напомним, что при умножении вектор на скаляр, мы умножаем каждый компонент отдельно на скаляр.Следовательно, единичный вектор равен равно четырем пятым, трем пятым.

Теперь рассмотрим несколько примеров. где нам нужно вычислить единичные векторы.

Рассмотрим вектор 𝐀 пять 𝐢 минус два 𝐣 минус четыре 𝐤. Единичный вектор в направлении такой же, как единичный вектор в направлении трех 𝐀?

Мы знаем, что вектор 𝐀 может быть переписать в виде пять, два отрицательных, четыре отрицательных.Нас также просят рассмотреть вектор три 𝐀. Это включает в себя умножение вектора 𝐀 скалярной или постоянной тройкой. Это предполагает умножение каждого компонент скалярной тройкой. Три, умноженные на пять, равны 15. Три, умноженные на два, равны отрицательная шестерка. И три умножить на минус четыре отрицательно 12. Это означает, что три 𝐀 равно до 15, отрицательное шесть, отрицательное 12.

Единичный вектор 𝐕 hat равен единицу на величину вектора 𝐕, умноженную на вектор.Это то же самое, что разделить вектор по величине. Мы знаем, что для расчета величины любого вектора, мы находим квадратный корень из суммы квадратов каждого вектора компонент. Величина вектора равна к квадратному корню из пяти в квадрате плюс минус два в квадрате плюс минус четыре в квадрате. Это равно квадратному корню из 25 плюс четыре плюс 16. Это упрощается до квадратного корня 45, что, в свою очередь, равно корню три пять.

Величина вектора равна трем. корень пять. Единичный вектор в направлении вектор 𝐀 равен единице над тремя корнями пять, умноженными на пять, два отрицательных, четыре отрицательных. Мы могли бы переписать это как вектор пять больше трех корень пять, минус два больше трех корень пять и минус четыре больше три корня пять.

Теперь рассмотрим единичный вектор в сторону трех 𝐀.Величина третьего вектора равна равно квадратному корню из 15 в квадрате плюс минус шесть в квадрате плюс минус 12 в квадрате. Это равно квадратному корню из 405, что, в свою очередь, упрощается до девяти корней пять. Величина третьего вектора равна равно девяти корню пять.

Здесь можно заметить, что это в три раза больше вектора. Это приводит нас к общему правило.Величина 𝑘𝐕 равна 𝑘 умножить на величину вектора 𝐕. Это означает, что три умноженные на модуль вектора 𝐀 равен трем, умноженным на три корня из пяти. Единичный вектор в направлении Таким образом, три 𝐀 равно единице из корня пять из девяти, умноженному на вектор 15, отрицательная шестерка, отрицательная 12.

Вычитая тройку из вектор дает нам три больше девяти корень пять, умноженный на вектор пять, отрицательный два, четыре отрицательных.Это то же самое, что и один из трех корень пять, умноженный на вектор пять, два отрицательных, четыре отрицательных. Таким образом, мы можем сделать вывод, что ответ — да, единичный вектор в направлении 𝐀 такой же, как единичный вектор в сторону трех 𝐀.

Это будет верно для любого вектора умноженный на положительный скаляр. Пока наше значение 𝑘 равно положительный, то единичный вектор в направлении 𝐀 будет таким же, как и единичный вектор в направлении 𝑘𝐀.

В следующем вопросе мы найдем единичный вектор в том же направлении, что и двумерный вектор.

Найдите единичный вектор в том же направление как вектор отрицательный три 𝐢 плюс пять 𝐣.

Мы знаем, что единичный вектор 𝐕 hat равна единице по модулю вектора 𝐕, умноженному на вектор, где величина двумерного вектора с компонентами 𝑎 и равна квадратный корень из в квадрате плюс в квадрате.

В этом вопросе у нас есть вектор с отрицательными тремя и пятью составляющими 𝐢 и. Величина этого вектора равна равняется квадратному корню из отрицательных трех в квадрате плюс пять в квадрате. Отрицательные три в квадрате равны девять, а пять в квадрате равно 25. Это означает, что величина вектор 𝐕 равен корню 34. Следовательно, единичный вектор равен равно единице над корнем 34, умноженным на минус три, пять.

При умножении любого вектора на скаляр, мы умножаем каждый отдельный компонент на скаляр. Это дает нам три отрицательных корень 34, пять над корнем 34. Мы можем рационализировать знаменатель единицы больше корня 34, умножив числитель и знаменатель на корень 34. Это означает, что единица больше корня 34 равна равен корню 34 больше 34. Это верно для любого радикала. Один над корнем 𝑎 равен корню 𝑎 над 𝑎.

Таким образом, мы можем переписать наши два компоненты как отрицательные три корня 34 над 34 и пять корней 34 над 34. Переписывая это в терминах 𝐢 и 𝐣, единичный вектор в том же направлении, что и вектор отрицательные три 𝐢 плюс пять 𝐣 равно отрицательным трем корням 34 над 34 𝐢 плюс пяти корням 34 над 34 𝐣.

В нашем последнем вопросе мы будем использовать комбинация скалярного умножения и единичных векторов.

Данный вектор 𝐀 равен двум, ноль, отрицательные два и вектор 𝐁 равен единице, отрицательный один, один, определяют единичный вектор в направлении два 𝐁 минус 𝐀.

Нашим первым шагом будет вычисление вектор два минус 𝐀. Мы делаем это, умножая вектор 𝐁 на скаляр два с последующим вычитанием вектора. При умножении вектора на скаляр, мы умножаем каждую из составляющих на скаляр. Следовательно, два 𝐁 равно двум, отрицательный два, два.

Нам нужно вычесть вектор два, ноль, два отрицательных. Мы делаем это, вычитая каждый компонент индивидуально.Два минус два равно нулю. Отрицательные два минус ноль равны отрицательные два. Наконец, два минус минус два — это равняется четырем, так как вычитание отрицательных двух равносильно сложению двух.

Нам нужно найти единичный вектор это. Мы знаем, что единичный вектор 𝐕 hat равна единице по величине 𝐕, умноженной на вектор. Величина два минус 𝐀 равна равняется квадратному корню из нуля в квадрате плюс отрицательные два в квадрате плюс четыре в квадрате.Это равно квадратному корню из 20, что упрощается до двух корень пять. Следовательно, единичный вектор равен к единице больше двух корень пять, умноженный на ноль, отрицательный два, четыре. Затем мы умножаем каждую отдельную компонент на один больше двух корень пять. Умножение этого на ноль дает нам нуль. Отрицательные два, умноженные на единицу, больше два корня пять равно отрицательному единице над корнем пять.

Рационализация знаменателя с помощью умножение числителя и знаменателя на корень пять дает нам отрицательный корень пять больше пяти.Наконец, умножая четыре на один над двумя корнем пять дает нам два корня пять над пятью. Единичный вектор в направлении два 𝐁 минус 𝐀 имеет компонент ноль, отрицательный корень пять больше пяти, а два корень пять больше пяти.

Теперь мы резюмируем ключ точки из этого видео. При умножении вектора на скаляр, мы умножаем каждую составляющую вектора на скаляр. Это означает, что 𝑘, умноженное на вектор 𝑎, 𝑏 равен вектору 𝑘𝑎, 𝑘𝑏.Это работает как в двух, так и в трех Габаритные размеры. Величина любого вектора равна обозначается столбцами абсолютного значения. Величина двумерного вектор 𝑎, 𝑏 равен квадратному корню из в квадрате плюс в квадрате.

Мы также обнаружили, что единичный вектор имеет величину один в том же направлении, что и данный вектор. Единичный вектор, обозначенный 𝐕 hat, равен равна единице по величине 𝐕, умноженной на вектор. Другой способ сказать это: единичный вектор равен данному вектору, деленному на его величину.

Понимание точечного произведения

Написано Робертом Dunlop
Microsoft DirectX MVP

Введение

Скалярное произведение — это величина, выражающая угловое соотношение между двумя векторы. В этой статье мы узнаем, как рассчитывается это значение, его математическое значение и несколько способов использования этой функции в 3D-приложения.

Вычисление скалярного произведения

Скалярное произведение — это скалярное значение, являющееся результатом операции двух векторы с одинаковым количеством компонентов. Для каждого из двух векторов A и B с компонентами n скалярное произведение рассчитывается как:

A B = A 1 B 1 + … + A n B n

Скалярное произведение, таким образом, представляет собой сумму произведений каждый компонент двух векторов.Например, если A и B были трехмерными векторами:

A B = A.x * B.x + A.y * B.y + A.z * B.z

Общая функция C ++ для реализации скалярного произведения на двух векторах с плавающей запятой любых размеров может выглядеть примерно так:

float dot_product (float * a, float * b, int size)
{
float dp = 0.0f;
для (int i = 0; i dp + = a [i] * b [i];
return dp;
}

Этот образец кода предоставлен исключительно для демонстрации общего функция для уточнения способа вычисления скалярного произведения; DirectX предоставляет несколько реализаций этой функции для вас, как вы увидите в дальнейшем, хотя, если вам нужно было написать свой собственный функция (например, при использовании C ++ без библиотек D3DX) вы, скорее всего, просто напишите отдельные функции для обработки часто используемых векторных типов (2D, 3D, 4D) как встроенный код.

Так что это значит?

Ранее мы говорили, что скалярное произведение представляет собой угловую зависимость между двумя векторами, и оставим на этом. Теперь рассмотрим подробнее что представляет собой это значение.

Допустим, у нас есть два вектора, A и B, как показано слева. В значения | A | и | B | представляют длины векторов A и B, соответственно, и Θ — угол между двумя векторами. Точечный продукт векторов A и B будет иметь следующее отношение к этим значения:

A B = | A | * | B | * cos ( Θ )

То есть скалярное произведение двух векторов будет быть равным косинусу угла между векторами, умноженному на длину каждый из векторов.

Угловая область точечного произведения:

Учитывая характеристики функции косинуса, мы можем вывести три возможных условия:

  1. Если A и B перпендикулярны (под углом 90 градусов к друг друга), результат скалярного произведения будет равен нулю, потому что cos ( Θ ) будет ноль.
  2. Если угол между A и B меньше 90 градусов, точечное произведение будет положительным (больше нуля), так как cos ( Θ ) будет положительным, а длины векторов всегда будут положительными значениями.
  3. Если угол между A и B больше 90 градусов, точечное произведение будет отрицательным (меньше нуля), так как cos ( Θ ) будет отрицательным, а длины векторов всегда положительные.

Угол от скалярного произведения единичных векторов

Приведенные выше характеристики верны для любых векторов ненулевой длины. Кроме того, существует особый случай, когда оба вектора являются единичными векторами, т.е. есть, векторы длиной один (1.0). В этом случае длина векторов не влияет на уравнение, упрощая до:

A B = | A | * | B | * cos ( )
A B = 1 * 1 * cos ( Θ )
A B = cos ( Θ )

В этом случае скалярное произведение равно косинус угла между векторами. Таким образом, угол между блоком векторы могут быть рассчитаны как:

Θ = acos (A B)

Угол от скалярного произведения неединичных векторов

Углы между неединичными векторами (векторами с длины не равны 1.0) можно вычислить либо предварительно нормализовав векторов, или путем деления скалярного произведения неединичных векторов на длину каждый вектор.

Точечное произведение вектора с самим собой

Взять скалярное произведение вектора на себя (т.е. A A) приводит к значению, равному квадрату длины вектора. Это знакомая часть уравнения расстояния, d = sqrt (x * x + y * y + z * z).

Проекция вектора на другой вектор

Если взять скалярное произведение единичного вектора A и второго вектора B любой ненулевой длины, результатом является длина вектора B, спроецированного в направлении вектора A (см. рисунок слева).Это используется разными способами, например, при столкновении отклик и преобразование векторов из одной системы координат в другую (это лежит в основе матричных преобразований).

Реализации DirectX

DirectX Graphics предоставляет несколько реализаций функции скалярного произведения:

2.3 Точечное произведение — Объем исчисления 3

Когда два ненулевых вектора помещаются в стандартное положение, будь то в двух измерениях или трех измерениях, они образуют угол между ними (рисунок 2.44). Точечное произведение позволяет найти меру этого угла. Это свойство является результатом того факта, что мы можем выразить скалярное произведение через косинус угла, образованного двумя векторами.

Проба

Поместите векторы u и v в стандартное положение и рассмотрите вектор v − uv − u (рисунок 2.45). Эти три вектора образуют треугольник с длинами сторон u‖, ‖v‖ и v − u‖.‖u‖, ‖v‖ и v − u‖.

Рис. 2.45 Длины сторон треугольника задаются модулями векторов, образующих треугольник.

Вспомните из тригонометрии, что закон косинусов описывает соотношение между длинами сторон треугольника и углом θ . Применение закона косинусов дает

‖V − u‖2 = ‖u‖2 + ‖v‖2−2‖u‖‖v‖cosθ.‖v − u‖2 = ‖u‖2 + ‖v‖2−2‖u‖‖v‖ cosθ.

Скалярное произведение позволяет переписать левую часть этого уравнения:

‖V − u‖2 = (v − u) · (v − u) = (v − u) · v− (v − u) · u = v · v − u · v − v · u + u · u = v · v − u · v − u · v + u · u = ‖v‖2−2u · v + ‖u‖2.‖v − u‖2 = (v − u) · (v − u) = ( v − u) · v− (v − u) · u = v · v − u · v − v · u + u · u = v · v − u · v − u · v + u · u = ‖v‖ 2−2u · v + ‖u‖2.

Подставляя в закон косинусов, получаем

‖V − u‖2 = ‖u‖2 + ‖v‖2−2‖u‖‖v‖cosθ‖v‖2−2u · v + ‖u‖2 = ‖u‖2 + ‖v‖2−2‖ u‖‖v‖cosθ − 2u · v = −2‖u‖‖v‖cosθu · v = ‖u‖‖v‖cosθ.‖v − u‖2 = ‖u‖2 + ‖v‖2−2‖ u‖‖v‖cosθ‖v‖2−2u · v + ‖u‖2 = ‖u‖2 + ‖v‖2−2‖u‖‖v‖cosθ − 2u · v = −2‖u‖‖v‖ cosθu · v = ‖u‖‖v‖cosθ.

Мы можем использовать эту форму скалярного произведения, чтобы найти меру угла между двумя ненулевыми векторами. Следующее уравнение преобразует уравнение 2.3 для определения косинуса угла:

cosθ = u · v‖u‖‖v‖.cosθ = u · v‖u‖‖v‖.

(2,5)

Используя это уравнение, мы можем найти косинус угла между двумя ненулевыми векторами. Поскольку мы рассматриваем наименьший угол между векторами, мы предполагаем, что 0 ° ≤θ≤180 ° 0 ° ≤θ≤180 ° (или 0≤θ≤π0≤θ≤π, если мы работаем в радианах). Обратный косинус уникален в этом диапазоне, поэтому мы можем определить меру угла θ.θ.

Пример 2.23

Нахождение угла между двумя векторами

Найдите угол между каждой парой векторов.

  1. i + j + k и 2 i j — 3 k
  2. 〈2,5,6〉 〈2,5,6〉 и 〈−2, −4,4〉 〈- 2, −4,4〉
Решение
  1. Чтобы найти косинус угла, образованного двумя векторами, подставьте компоненты векторов в уравнение 2.5:
    cosθ = (i + j + k) · (2i − j − 3k) ‖i + j + k‖ · ‖2i − j − 3k‖ = 1 (2) + (1) (- 1) + (1) ( −3) 12 + 12 + 1222 + (- 1) 2 + (- 3) 2 = −2314 = −242. Cosθ = (i + j + k) · (2i − j − 3k) ‖i + j + k ‖ · ‖2i − j − 3k‖ = 1 (2) + (1) (- 1) + (1) (- 3) 12 + 12 + 1222 + (- 1) 2 + (- 3) 2 = −2314 = −242.
    Следовательно, θ = arccos − 242θ = arccos − 242 рад.
  2. Начните с определения значения косинуса угла между векторами:
    cosθ = 〈2,5,6〉 · 〈−2, −4,4〉 ‖ 〈2,5,6〉 ‖ · 〈−2, −4,4〉 ‖ = 2 (−2) + (5) (−4) + (6) (4) 22 + 52 + 62 (−2) 2 + (- 4) 2 + 42 = 06536 = 0. cosθ = 〈2,5,6〉 · 〈−2, −4 , 4〉 ‖ 〈2,5,6〉 ‖ · ‖ 〈−2, −4,4〉 ‖ = 2 (−2) + (5) (- 4) + (6) (4) 22 + 52 + 62 (−2) 2 + (- 4) 2 + 42 = 06536 = 0.
    Теперь cosθ = 0cosθ = 0 и 0≤θ≤π, 0≤θ≤π, поэтому θ = π / 2.θ = π / 2.

КПП 2.23

Найдите угол в радианах, образованный векторами a = 〈1,2,0〉 a = 〈1,2,0〉 и b = 〈2,4,1〉.б = 〈2,4,1〉. Округлить до ближайшей сотой.

Угол между двумя векторами может быть острым (0 Рис. 2.46 (a) У острого угла 0 Теорема 2.5

Ортогональные векторы

Ненулевые векторы u и v являются ортогональными векторами тогда и только тогда, когда u · v = 0.u · v = 0.

Проба

Пусть u и v ненулевые векторы, и пусть θθ обозначает угол между ними.Сначала предположим, что u · v = 0. u · v = 0. Тогда

‖U‖‖v‖cosθ = 0.‖u‖‖v‖cosθ = 0.

Однако u‖ ≠ 0‖u‖ ≠ 0 и ‖v‖ ≠ 0, ‖v‖ ≠ 0, поэтому мы должны иметь cosθ = 0. cosθ = 0. Следовательно, θ = 90 °, θ = 90 °, и векторы ортогональны.

Теперь предположим, что u и v ортогональны. Тогда θ = 90 ° θ = 90 ° и имеем

u · v = u‖‖v‖cosθ = ‖u‖‖v‖cos90 ° = ‖u‖‖v‖ (0) = 0. u · v = ‖u‖‖v‖cosθ = ‖u‖‖v ‖Cos90 ° = ‖u‖‖v‖ (0) = 0.

Термины ортогональный , перпендикулярный и нормальный каждый указывают, что математические объекты пересекаются под прямым углом.Использование каждого термина определяется главным образом его контекстом. Мы говорим, что векторы ортогональны, а прямые перпендикулярны. Термин нормальный используется чаще всего при измерении угла, образованного плоскостью или другой поверхностью.

Пример 2.24

Идентификация ортогональных векторов

Определите, являются ли векторы p = 〈1,0,5〉 p = 〈1,0,5〉 и q = 〈10,3, −2〉 q = 〈10,3, −2〉 ортогональными векторами.

Решение

Используя определение, нам нужно только проверить скалярное произведение векторов:

p · q = 1 (10) + (0) (3) + (5) (- 2) = 10 + 0−10 = 0.p · q = 1 (10) + (0) (3) + (5) (- 2) = 10 + 0−10 = 0.

Поскольку p · q = 0, p · q = 0, векторы ортогональны (рисунок 2.47).

Рис. 2.47 Векторы p и q образуют прямой угол, когда их начальные точки совмещены.

КПП 2.24

, для которого значение x равно p = 〈2,8, −1〉 p = 〈2,8, −1〉, ортогонально q = 〈x, −1,2〉? Q = 〈x, −1, 2〉?

Пример 2.25

Измерение угла, образованного двумя векторами

Пусть v = 〈2,3,3〉 .v = 〈2,3,3〉.Найдите размеры углов, образованных следующими векторами.

  1. v и i
  2. v и j
  3. v и k
Решение
  1. Пусть α будет углом, образованным v и i:
    cosα = v · i‖v‖ · ‖i‖ = 〈2,3,3〉 · 〈1,0,0〉 22 + 32 + 321 = 222. cosα = v · i‖v‖ · ‖i‖ = 〈 2,3,3〉 · 〈1,0,0〉 22 + 32 + 321 = 222.
    α = arccos222≈1,130рад. α = arccos222≈1.130рад.
  2. Пусть β представляет угол, образованный v и j :
    cosβ = v · j‖v‖ · ‖j‖ = 〈2,3,3〉 · 〈0,1,0〉 22 + 32 + 321 = 322. cosβ = v · j‖v‖ · ‖j‖ = 〈 2,3,3〉 · 〈0,1,0〉 22 + 32 + 321 = 322.
    β = arccos322≈0,877рад. β = arccos322≈0,877рад.
  3. Пусть γ представляет угол, образованный v и k :
    cosγ = v · k‖v‖ · ‖k‖ = 〈2,3,3〉 · 〈0,0,1〉 22 + 32 + 321 = 322. cosγ = v · k‖v‖ · ‖k‖ = 〈 2,3,3〉 · 〈0,0,1〉 22 + 32 + 321 = 322.
    γ = arccos322≈0,877рад. γ = arccos322≈0,877рад.

КПП 2.25

Пусть v = 〈3, −5,1〉 .v = 〈3, −5,1〉. Найдите меру углов, образованных каждой парой векторов.

  1. v и i
  2. v и j
  3. v и k

Угол, который вектор образует с каждой из координатных осей, называемый углом направления, очень важен в практических вычислениях, особенно в такой области, как инженерия.Например, в космонавтике угол запуска ракеты должен определяться очень точно. Очень маленькая ошибка в угле может привести к тому, что ракета отклонится от курса на сотни миль. Углы направления часто вычисляются с помощью скалярного произведения и косинусов углов, называемых направляющими косинусами. Поэтому мы определяем как эти углы, так и их косинусы.

Определение

Углы, образованные ненулевым вектором и осями координат, называются углами направления вектора (рисунок 2.48). Косинусы для этих углов называются направляющими косинусами.

Рис. 2.48 Угол α образован вектором v и единичным вектором i . Угол β образован вектором v и единичным вектором j . Угол γ образован вектором v и единичным вектором k .

В примере 2.25 направляющие косинусы v = 〈2,3,3〉 v = 〈2,3,3〉 равны cosα = 222, cosα = 222, cosβ = 322, cosβ = 322 и cosγ = 322.cosγ = 322. Углы направления v равны α = 1,130рад, α = 1,130рад, β = 0,877рад, β = 0,877рад и γ = 0,877рад. Γ = 0,877рад.

До сих пор мы сосредоточились в основном на векторах, связанных с силой, движением и положением в трехмерном физическом пространстве. Однако векторы часто используются более абстрактно. Например, предположим, что продавец фруктов продает яблоки, бананы и апельсины. В определенный день он продает 30 яблок, 12 бананов и 18 апельсинов. Он может использовать вектор количества q = 〈30,12,18〉, q = 〈30,12,18〉, чтобы представить количество фруктов, которые он продал в тот день.Точно так же он может захотеть использовать вектор цен, p = 〈0.50,0.25,1〉, p = 〈0.50,0.25,1〉, чтобы указать, что он продает свои яблоки по 50 каждое, бананы по 25 ¢ каждый и апельсины по 1 доллару за штуку. В этом примере, хотя мы по-прежнему можем изобразить эти векторы, мы не интерпретируем их как буквальные представления положения в физическом мире. Мы просто используем векторы, чтобы отслеживать отдельные фрагменты информации о яблоках, бананах и апельсинах.

Эта идея может показаться немного странной, но если мы просто будем рассматривать векторы как способ упорядочивания и хранения данных, мы обнаружим, что они могут быть довольно мощным инструментом.Возвращаясь к продавцу фруктов, давайте подумаем о скалярном произведении q · p.q · p. Мы вычисляем это, умножая количество проданных яблок (30) на цену за яблоко (50 центов), количество проданных бананов на цену за банан и количество проданных апельсинов на цену за апельсин. Затем мы складываем все эти значения вместе. Итак, в этом примере скалярный продукт сообщает нам, сколько денег продавец фруктов имел от продаж в этот конкретный день.

Когда мы используем векторы в более общем смысле, нет причин ограничивать количество компонентов тремя.Что, если продавец фруктов решит начать продавать грейпфрут? В этом случае он хотел бы использовать четырехмерные векторы количества и цен для представления количества проданных яблок, бананов, апельсинов и грейпфрутов и их удельных цен. Как и следовало ожидать, для вычисления скалярного произведения четырехмерных векторов мы просто складываем произведения компонентов, как и раньше, но в сумме четыре члена вместо трех.

Пример 2.26

Использование векторов в экономическом контексте

AAA Party Supply Store продает приглашения, праздничные сувениры, украшения и предметы общественного питания, такие как бумажные тарелки и салфетки.Когда AAA покупает свой инвентарь, он платит 25 центов за упаковку за приглашения и вечеринки. Украшения стоят 50 центов AAA каждое, а предметы общественного питания — 20 центов за упаковку. AAA продает приглашения по цене 2,50 доллара за пакет, а сувениры для вечеринок по цене 1,50 доллара за пакет. Украшения продаются по 4,50 доллара за штуку, а предметы общественного питания — по 1,25 доллара за упаковку.

В течение мая AAA Party Supply Store продает 1258 приглашений, 342 праздничных подарка, 2426 украшений и 1354 предмета общественного питания. Используйте векторы и точечные произведения, чтобы подсчитать, сколько денег AAA заработало на продажах в мае.Какую прибыль принес магазин?

Решение

Векторы затрат, цены и количества равны

. c = 〈0,25,0,25,0,50,0,20〉 p = 〈2,50,1,50,4,50,1,25〉 q = 〈1258,342,2426,1354〉. c = 〈0,25,0,25,0,50,0,20〉 p = 〈2.50, 1,50,4,50,1,25〉 q = 〈1258,342,2426,1354〉.

продаж AAA в мае можно рассчитать с помощью скалярного произведения p · q.p · q. У нас

p · q = 〈2,50,1,50,4,50,1,25〉 · 〈1258,342,2426,1354〉 = 3145 + 513 + 10917 + 1692,5 = 16267,5.p · q = 〈2,50,1,50,4,50,1,25〉 · 〈1258 , 342,2426,1354〉 = 3145 + 513 + 10917 + 1692.5 = 16267,5.

Итак, AAA заработала 16 267,50 долларов в течение мая.

Чтобы рассчитать прибыль, мы должны сначала подсчитать, сколько AAA заплатило за проданные товары. Мы используем скалярное произведение c · qc · q, чтобы получить

c · q = 〈0,25,0,25,0,50,0,20〉 · 〈1258,342,2426,1354〉 = 314,5 + 85,5 + 1213 + 270,8 = 1883,8.c · q = 〈0,25,0,25,0,50,0,20〉 · 〈1258 , 342, 2426, 1354〉 = 314,5 + 85,5 + 1213 + 270,8 = 1883,8.

Итак, AAA заплатила 1883,80 доллара за проданные товары. Таким образом, их прибыль равна

p · q − c · q = 16267,5−1883,8 = 14383,7.p · q − c · q = 16267.5−1883,8 = 14383,7.

Таким образом, магазин AAA Party Supply в мае заработал 14 383,70 долларов.

КПП 2.26

1 июня магазин AAA Party Supply решил повысить цену, которую они взимают за праздничные сувениры, до 2 долларов за упаковку. Они также сменили поставщиков для своих приглашений и теперь могут покупать приглашения всего за 10 центов за упаковку. Все остальные затраты и цены остаются прежними. Если AAA продает 1408 приглашений, 147 сувениров для вечеринок, 2112 украшений и 1894 предмета общественного питания в июне, используйте векторы и точечные продукты для расчета их общих продаж и прибыли за июнь.

Мягкое введение в векторные нормы в машинном обучении

Последнее обновление 9 августа 2019 г.

Расчет длины или величины векторов часто требуется либо непосредственно как метод регуляризации в машинном обучении, либо как часть более широких векторных или матричных операций.

В этом руководстве вы откроете для себя различные способы вычисления длины или величины векторов, которые называются векторной нормой.

После прохождения этого руководства вы будете знать:

  • Норма L1, вычисляемая как сумма абсолютных значений вектора.
  • Норма L2, которая вычисляется как квадратный корень из суммы квадратов значений вектора.
  • Максимальная норма, вычисляемая как максимальные значения вектора.

Начните свой проект с моей новой книги «Линейная алгебра для машинного обучения», включающей пошаговых руководств и файлов исходного кода Python для всех примеров.

Приступим.

  • Обновление март / 2018 : исправлена ​​опечатка в уравнении максимальной нормы.
  • Обновление сентябрь 2018 г. : исправлена ​​опечатка, связанная с размером определенных векторов.

Мягкое введение в векторные нормы в машинном обучении
Фото Cosimo, некоторые права защищены.

Обзор учебного пособия

Это руководство разделено на 4 части; их:

  1. Вектор Норма
  2. Вектор L1 Норма
  3. Вектор L2 Норма
  4. Vector Max Norm

Нужна помощь с линейной алгеброй для машинного обучения?

Пройдите бесплатный 7-дневный ускоренный курс по электронной почте (с образцом кода).

Нажмите, чтобы зарегистрироваться, а также получите бесплатную электронную версию курса в формате PDF.

Загрузите БЕСПЛАТНЫЙ мини-курс

Вектор Норма

Расчет размера или длины вектора часто требуется либо напрямую, либо как часть более широкой векторной или векторно-матричной операции.

Длина вектора называется векторной нормой или величиной вектора.

Длина вектора — это неотрицательное число, которое описывает протяженность вектора в пространстве и иногда называется величиной вектора или нормой.

— Стр. 112, Руководство по линейной алгебре без ерунды, 2017

Длина вектора всегда является положительным числом, за исключением вектора всех нулевых значений. Он вычисляется с использованием некоторой меры, которая суммирует расстояние вектора от начала векторного пространства. Например, начало векторного пространства для вектора с 3 элементами — (0, 0, 0).

Обозначения используются для представления векторной нормы в более широких вычислениях, и тип вычисления векторной нормы почти всегда имеет свое собственное уникальное обозначение.1.

Обозначение L1 нормы вектора: || v || 1, где 1 — нижний индекс. Таким образом, эту длину иногда называют нормой такси или нормой Манхэттена.

Норма L1 вычисляется как сумма абсолютных значений вектора, где абсолютное значение скаляра использует обозначение | a1 |. Фактически, норма — это вычисление манхэттенского расстояния от начала векторного пространства.

|| v || 1 = | a1 | + | a2 | + | a3 |

|| v || 1 = | a1 | + | a2 | + | a3 |

Норма L1 вектора может быть вычислена в NumPy с помощью функции norm () с параметром для указания порядка нормы, в данном случае 1.

# l1 норма вектора из массива импорта numpy из нормы импорта numpy.linalg a = массив ([1, 2, 3]) печать (а) l1 = norm (a, 1) печать (l1)

# l1 norm вектора

из массива импорта numpy

из нормы импорта numpy.linalg

a = array ([1, 2, 3])

print (a)

l1 = norm (a, 1)

печать (l1)

Сначала определяется вектор размером 1 × 3, затем вычисляется норма L1 вектора.2.

Обозначение L2 нормы вектора: || v || 2, где 2 — нижний индекс.

Норма L2 вычисляет расстояние векторной координаты от начала векторного пространства. 2)

Норма L2 вектора может быть вычислена в NumPy с помощью функции norm () с параметрами по умолчанию.

# l2 норма вектора из массива импорта numpy из нормы импорта numpy.linalg a = массив ([1, 2, 3]) печать (а) l2 = норма (а) печать (l2)

# l2 norm вектора

из массива импорта numpy

из numpy.linalg import norm

a = array ([1, 2, 3])

print (a)

l2 = norm (a)

печать (l2)

Сначала определяется вектор размером 1 × 3, затем вычисляется L2-норма вектора.

При выполнении примера сначала печатается определенный вектор, а затем его норма L2.

Как и норма L1, норма L2 часто используется при подборе алгоритмов машинного обучения в качестве метода регуляризации, например метод, позволяющий сохранить небольшие коэффициенты модели и, в свою очередь, сделать модель менее сложной.

Безусловно, норма L2 используется в машинном обучении чаще, чем другие векторные нормы.

Vector Max Norm

Длину вектора можно вычислить с использованием максимальной нормы, также называемой максимальной нормой.inf, где inf — это верхний индекс, который может быть представлен символом бесконечности. Обозначение max norm: || x || inf, где inf — нижний индекс.

Максимальная норма вычисляется как возвращаемое максимальное значение вектора, отсюда и название.

|| v || inf = max (| a1 |, | a2 |, | a3 |)

|| v || inf = max (| a1 |, | a2 |, | a3 |)

Максимальную норму вектора можно вычислить в NumPy с помощью функции norm () с параметром порядка, установленным на inf.

# максимальная норма вектора из numpy import inf из массива импорта numpy из нормы импорта numpy.linalg a = массив ([1, 2, 3]) печать (а) maxnorm = norm (a, inf) печать (maxnorm)

# максимальная норма вектора

из numpy import inf

из массива импорта numpy

из numpy.linalg import norm

a = array ([1, 2, 3])

print (a)

maxnorm = norm (a, inf)

отпечаток (maxnorm)

Сначала определяется вектор размером 1 × 3, затем вычисляется максимальная норма вектора.

При выполнении примера сначала печатается определенный вектор, а затем максимальная норма вектора.

Максимальная норма также используется в качестве регуляризации в машинном обучении, например, для весов нейронной сети, называемой регуляризацией максимальной нормы.

Расширения

В этом разделе перечислены некоторые идеи по расширению учебника, которые вы, возможно, захотите изучить.

  • Создайте 5 примеров, используя каждую операцию, используя свои собственные данные.
  • Выполните каждую матричную операцию вручную для матриц, определенных как списки списков.
  • Найдите статьи по машинному обучению и найдите по одному примеру каждой используемой операции.

Если вы изучите какое-либо из этих расширений, я хотел бы знать.

Дополнительная литература

Этот раздел предоставляет дополнительные ресурсы по теме, если вы хотите углубиться.

Книги

API

Статьи

Сводка

В этом руководстве вы открыли для себя различные способы вычисления длины или величины векторов, которые называются векторной нормой.

В частности, вы выучили:

  • Норма L1, вычисляемая как сумма абсолютных значений вектора.
  • Норма L2, которая вычисляется как квадратный корень из суммы квадратов значений вектора.
  • Максимальная норма, вычисляемая как максимальные значения вектора.

Есть вопросы?
Задайте свои вопросы в комментариях ниже, и я постараюсь ответить.

Познакомьтесь с линейной алгеброй для машинного обучения!

Развивайте рабочее понимание линейной алгебры

…пишите строки кода на Python

Узнайте, как это сделать, в моей новой электронной книге:
Линейная алгебра для машинного обучения

Он предоставляет руководств для самообучения по таким темам, как:
векторные нормы, умножение матриц, тензоры, собственное разложение, SVD, PCA и многое другое …

И наконец разобраться в математике данных

Пропустить академики. Только результаты.

Посмотрите, что внутри

13.8: Работа и скалярное произведение

Мы введем векторную операцию, называемую скалярным произведением или «скалярным произведением», которая берет любые два вектора и генерирует скалярную величину (число).Мы увидим, что физическая концепция работы может быть математически описана скалярным произведением между силой и векторами смещения.

Скалярное произведение

Пусть \ (\ overrightarrow {\ mathbf {A}} \) и \ (\ overrightarrow {\ mathbf {B}} \) два вектора. Поскольку любые два неколлинеарных вектора образуют плоскость, мы определяем угол θ как угол между векторами \ (\ overrightarrow {\ mathbf {A}} \) и \ (\ overrightarrow {\ mathbf {B}} \) как показано на рисунке 13.9. Обратите внимание, что θ может изменяться от 0 до \ (\ pi \).

Рисунок 13.9 Скалярная геометрия произведения.

Скалярное произведение \ (\ overrightarrow {\ mathbf {A}} \ cdot \ overrightarrow {\ mathbf {B}} \) векторов \ (\ overrightarrow {\ mathbf {A}} \) и \ (\ overrightarrow { \ mathbf {B}} \) определяется как произведение величины векторов \ (\ overrightarrow {\ mathbf {A}} \) и \ (\ overrightarrow {\ mathbf {B}} \) с косинусом угол θ между двумя векторами:

\ [\ overrightarrow {\ mathbf {A}} \ cdot \ overrightarrow {\ mathbf {B}} = A B \ cos (\ theta) \]

, где \ (A = | \ overrightarrow {\ mathbf {A}} | \ text {и} B = \ mid \ overrightarrow {\ mathbf {B}} \) представляют величину \ (\ overrightarrow {\ mathbf {A }} \) и \ (\ overrightarrow {\ mathbf {B}} \) соответственно.{1/2} \]

Мы можем дать геометрическую интерпретацию скалярному произведению, записав определение как

\ [\ overrightarrow {\ mathbf {A}} \ cdot \ overrightarrow {\ mathbf {B}} = (A \ cos (\ theta)) B \]

В этой формулировке термин Acosθ — это проекция вектора \ (\ overrightarrow {\ mathbf {B}} \) в направлении вектора \ (\ overrightarrow {\ mathbf {B}} \). Эта проекция показана на рисунке 13.10a. Таким образом, скалярное произведение — это произведение проекции длины \ (\ overrightarrow {\ mathbf {A}} \) в направлении \ (\ overrightarrow {\ mathbf {B}} \) на длину \ ( \ overrightarrow {\ mathbf {B}} \).Обратите внимание, что мы также можем записать скалярное произведение как

\ [\ overrightarrow {\ mathbf {A}} \ cdot \ overrightarrow {\ mathbf {B}} = A (B \ cos (\ theta)) \]

Теперь член \ (B \ cos (\ theta) \) — это проекция вектора \ (\ overrightarrow {\ mathbf {B}} \) в направлении вектора \ (\ overrightarrow {\ mathbf {A}) } \), как показано на рисунке 13.10b. С этой точки зрения скалярное произведение — это произведение проекции длины \ (\ overrightarrow {\ mathbf {B}} \) в направлении \ (\ overrightarrow {\ mathbf {A}} \) на длину из \ (\ overrightarrow {\ mathbf {A}} \).

Рис. 13.10 (a) и (b) Проекция векторов и скалярное произведение

Из нашего определения скалярного произведения мы видим, что скалярное произведение двух векторов, перпендикулярных друг другу, равно нулю, поскольку угол между векторами равен \ ( \ pi / 2 \) и \ (\ cos (\ pi / 2) = 0 \).

Мы можем вычислить скалярное произведение двух векторов в декартовой системе координат следующим образом. Рассмотрим два вектора \ (\ overrightarrow {\ mathbf {A}} = A_ {x} \ hat {\ mathbf {i}} + A_ {y} \ hat {\ mathbf {j}} + A_ {z} \ hat { \ mathbf {k}} \) и \ (\ overrightarrow {\ mathbf {B}} = B_ {x} \ hat {\ mathbf {i}} + B_ {y} \ hat {\ mathbf {j}} + B_ {z} \ hat {\ mathbf {k}} \).Напомним, что

\ [\ begin {array} {l}
\ hat {\ mathbf {i}} \ cdot \ hat {\ mathbf {i}} = \ hat {\ mathbf {j}} \ cdot \ hat {\ mathbf { j}} = \ hat {\ mathbf {k}} \ cdot \ hat {\ mathbf {k}} = 1 \\
\ hat {\ mathbf {i}} \ cdot \ hat {\ mathbf {j}} = \ hat {\ mathbf {j}} \ cdot \ hat {\ mathbf {k}} = \ hat {\ mathbf {i}} \ cdot \ hat {\ mathbf {k}} = 0
\ end {array} \ ]

Скалярное произведение между \ (\ overrightarrow {\ mathbf {A}} \) и \ (\ overrightarrow {\ mathbf {B}} \) тогда равно

\ [\ overrightarrow {\ mathbf {A}} \ cdot \ overrightarrow {\ mathbf {B}} = A_ {x} B_ {x} + A_ {y} B_ {y} + A_ {z} B_ {z} \]

Производная по времени скалярного произведения двух векторов равна

. {2} = \ frac {d} {dt} (\ overrightarrow {\ mathbf {A}} \ cdot \ overrightarrow {\ mathbf {A}}) = \ left (\ frac {d} {dt} \ overrightarrow {\ mathbf {A}} \ right) \ cdot \ overrightarrow {\ mathbf {A}} + \ overrightarrow {\ mathbf {A}} \ cdot \ left (\ frac { d} {dt} \ overrightarrow {\ mathbf {A}} \ right) = 2 \ left (\ frac {d} {dt} \ overrightarrow {\ mathbf {A}} \ right) \ cdot \ overrightarrow {\ mathbf { A}} \]

Кинетическая энергия и скалярное произведение

Для объекта, находящегося в трехмерном движении, скорость объекта в декартовых компонентах задается как \ (\ overrightarrow {\ mathbf {v}} = v_ {x} \ hat {\ mathbf {i}} + v_ {y } \ hat {\ mathbf {j}} + v_ {z} \ hat {\ mathbf {k}} \).{2} \ right) \]

Работа и скалярное произведение

Работа — важный физический пример математической операции взятия скалярного произведения между двумя векторами. Напомним, что когда на тело действует постоянная сила и точка приложения силы претерпевает смещение вдоль оси x, только составляющая силы вдоль этого направления вносит вклад в работу,

\ [W = F_ {x} \ Delta x \]

Предположим, мы тянем тело по горизонтальной поверхности с силой \ (\ overrightarrow {\ mathbf {F}} \).Выберите координаты так, чтобы горизонтальное направление было осью x, а сила \ (\ overrightarrow {\ mathbf {F}} \) образовывала угол \ (\ beta \) с положительным направлением x. На рисунке 13.11 показан вектор силы \ (\ overrightarrow {\ mathbf {F}} = F_ {x} \ hat {\ mathbf {i}} + F_ {y} \ hat {\ mathbf {j}} \) и вектор смещения точки приложения силы \ (\ Delta \ overrightarrow {\ mathbf {x}} = \ Delta x \ hat {\ mathbf {i}} \). Обратите внимание, что \ (\ Delta \ overrightarrow {\ mathbf {x}} = \ Delta x \ hat {\ mathbf {i}} \) является компонентом смещения и, следовательно, может быть больше, равно или меньше нуля (но для ясности на рисунке показано больше нуля).Скалярное произведение между вектором силы \ (\ overrightarrow {\ mathbf {F}} \) и вектором смещения \ (\ Delta \ overrightarrow {\ mathbf {x}} \) равно

.

\ [\ overrightarrow {\ mathbf {F}} \ cdot \ Delta \ overrightarrow {\ mathbf {x}} = \ left (F_ {x} \ hat {\ mathbf {i}} + F_ {y} \ hat { \ mathbf {j}} \ right) \ cdot (\ Delta x \ hat {\ mathbf {i}}) = F_ {x} \ Delta x \]

Рисунок 13.11 Векторы силы и смещения

Тогда работа, совершаемая силой, равна

.

\ [\ Delta W = \ overrightarrow {\ mathbf {F}} \ cdot \ Delta \ overrightarrow {\ mathbf {x}} \]

Обычно угол \ (\ beta \) принимает значения в диапазоне \ (- \ pi \ leq \ beta \ leq \ pi \) (на рисунке 13.11, \ (0 \ leq \ beta \ leq \ pi / 2 \)). Поскольку x -компонент силы равен \ (F_ {x} = F \ cos (\ beta) \), где \ (F = | \ overrightarrow {\ mathbf {F}} | \) обозначает величину \ (\ overrightarrow {\ mathbf {F}} \), работа, выполненная силой, равна

\ [W = \ overrightarrow {\ mathbf {F}} \ cdot \ Delta \ overrightarrow {\ mathbf {x}} = (F \ cos (\ beta)) \ Delta x \]

Пример 13.10 Объект, скользящий по наклонной плоскости

Объект массой m = 4,0 кг, вылетая из состояния покоя, скользит по наклонной плоскости длиной l = 3.{\ circ} \) на землю. Коэффициент кинетического трения равен \ (\ mu_ {k} = 0,2 \). (А) Какую работу совершает каждая из трех сил, когда объект скользит по наклонной плоскости? (б) Для каждой силы работа, совершаемая силой, положительна или отрицательна? в) Какова сумма работы, проделанной тремя силами? Это положительно или отрицательно?

Решение: (a) и (b) Выберите систему координат с началом координат в верхней части наклонной плоскости и положительным направлением оси x, указывающим вниз по наклонной плоскости, и положительным направлением оси y, указывающим вверх вправо, как показано на Рисунок 13.12. Пока объект скользит по наклонной плоскости, на него действуют три равномерные силы: сила тяжести, направленная вниз и имеющая величину \ (F_ {g} = mg \), нормальная сила N, перпендикулярная поверхности. наклонной плоскости, и сила трения, которая противодействует движению и равна по величине \ (f_ {k} = \ mu_ {k} N \). Диаграмма силы на объекте представлена ​​на рисунке 13.13.

Рисунок 13.12 Система координат объекта, скользящего по наклонной плоскости Рисунок 13.13 Диаграмма силы свободного тела для объекта

Для расчета работы нам необходимо определить, какие силы имеют составляющую в направлении смещения. Только составляющая гравитационной силы в положительном x-направлении \ (F_ {g x} = m g \ sin \ theta \) и сила трения направлены вдоль смещения и, следовательно, вносят свой вклад в работу. Нам нужно использовать Второй закон Ньютона, чтобы определить величину нормальной силы. Поскольку объект вынужден двигаться в положительном направлении по оси x, \ (a_ {y} = 0 \), второй закон Ньютона в \ (\ hat {\ mathbf {j}} \) — направлении \ (Nm g \ cos \ theta = 0 \). {N} = mg \ cos \ theta \ hat {\ mathbf {j}} \).{\ circ} \ right) \ right) = 38,4 \ mathrm {J}. \ right.
\ end {array} \]

12.3 Точечное произведение

Вот вопрос, ответ на который оказывается очень полезным: Учитывая два векторы, какой угол между ними?

Может быть не сразу понятно, что вопрос имеет смысл, но нетрудно превратить это в вопрос, который имеет значение. Поскольку векторы имеют нет позиции, мы, как обычно, можем размещать векторы везде, где мы как. Если два вектора расположены «хвост к хвосту», теперь имеется разумная интерпретация вопроса: мы ищем меру наименьший угол между двумя векторами в плоскости, в которой они лежат.Рисунок 12.3.1 иллюстрирует ситуацию.

Рисунок 12.3.1. Угол между векторами $ {\ bf A} $ и $ {\ bf B} $.

Поскольку угол $ \ theta $ лежит в треугольнике, мы можем вычислить его, используя немного тригонометрии, а именно закон косинусов. Длины сторон треугольник на рисунке 12. 2) \ cr & = 2a_1b_1 + 2a_2b_2 + 2a_3b_3 \ cr | {\ bf A} || {\ bf B} | \ cos \ theta & = a_1b_1 + a_2b_2 + a_3b_3 \ cr \ cos \ theta & = (a_1b_1 + a_2b_2 + a_3b_3) / (| {\ bf A} || {\ bf B} |) \ cr } $$ Итак, немного простой арифметики с координатами $ \ bf A $ и $ \ bf B $ позволяет нам вычислить косинус угла между ними.Если необходимо, мы можем использовать арккосинус, чтобы получить $ \ theta $, но во многих проблемы $ \ cos \ theta $ оказывается всем, что нам действительно нужно.

Числитель дроби, которая дает нам $ \ cos \ theta $, оказывается лот, поэтому мы даем ему имя и более компактные обозначения: мы называем его скалярное произведение и запишите его как $$ {\ bf A} \ cdot {\ bf B} = a_1b_1 + a_2b_2 + a_3b_3. $$ Это тот же символ, который мы используем для обычного умножения, но там никогда не должно быть путаницы; вы можете сказать из контекста, были ли мы являются «умножающими» векторами или числами.(Мы также можем использовать точку для скалярное умножение: $ a \ cdot {\ bf V} = a {\ bf V} $; опять же ясно что имеется в виду из контекста.)

Пример 12.3.1 Найдите угол между векторами $ {\ bf A} = \ langle 1,2,1 \ rangle $ и $ {\ bf B} = \ langle 3,1, -5 \ rangle $. Мы знаем это $ \ cos \ theta = {\ bf A} \ cdot {\ bf B} / (| {\ bf A} || {\ bf B} |) = (1 \ cdot3 + 2 \ cdot1 + 1 \ cdot (-5)) / (| {\ bf A} || {\ bf B} |) = 0 $, поэтому $ \ theta = \ pi / 2 $, то есть векторы перпендикулярны. $ \ квадрат $

Пример 12.3.2 Найдите угол между векторами $ {\ bf A} = \ langle 3,3,0 \ rangle $ и $ {\ bf B} = \ langle 1,0,0 \ rangle $.2}) $, который не определен. С другой стороны, обратите внимание, что, поскольку $ {\ bf A} \ cdot {\ bf 0} = 0 $, это выглядит сначала, как если бы $ \ cos \ theta $ был равен нулю, что, как мы видели, означает что векторы перпендикулярны; только когда мы замечаем, что знаменатель также равен нулю. Один из способов «исправить» это означает принятие соглашения о том, что нулевой вектор $ {\ bf 0} $ перпендикулярно всем векторам; то в общем можно сказать, что если $ {\ bf A} \ cdot {\ bf B} = 0 $, $ \ bf A $ и $ \ bf B $ перпендикулярны. $ \ квадрат $

Обобщая примеры, отметим следующие полезные факты:

    1.Если $ \ bf A $ параллельно или антипараллельно $ \ bf B $, то $ {\ bf A} \ cdot {\ bf B} / (| {\ bf A} || {\ bf B} |) = \ pm1 $, и наоборот, если $ {\ bf A} \ cdot {\ bf B} / (| {\ bf A} || {\ bf B} |) = 1 $, $ \ bf A $ и $ \ bf B $ параллельны, а если $ {\ bf A} \ cdot {\ bf B} / (| {\ bf A} || {\ bf B} |) = — 1 $, $ \ bf A $ и $ \ bf B $ антипараллельны. (Векторы параллельный если они указывают в одном направлении, антипараллельный если они указывают в противоположных направлениях.)

    2. Если $ \ bf A $ перпендикулярно $ \ bf B $, то $ {\ bf A} \ cdot {\ bf B} / (| {\ bf A} || {\ bf B} |) = 0 $, и наоборот, если $ {\ bf A} \ cdot {\ bf B} / (| {\ bf A} || {\ bf B} |) = 0 $, тогда $ \ bf A $ и $ \ bf B $ перпендикулярны.

Учитывая два вектора, часто бывает полезно найти выступ одного вектора на другой, потому что это оказывается важным значение во многих обстоятельствах. 2} {\ bf B}.2} {\ bf B} $$ антипараллельна $ \ bf B $, а его длина равна $$ \ left | {{\ bf A} \ cdot {\ bf B} \ over | {\ bf B} |} \ right |. $$ Итак, в общем случае скалярная проекция $ \ bf A $ на $ \ bf B $ может быть положительным или отрицательным. Если отрицательный, это означает, что вектор проекции антипараллельный в $ \ bf B $ и что длина вектора проекции равна абсолютное значение скалярной проекции. Конечно, вы также можете вычислить длину вектора проекции как обычно, применяя формула расстояния до вектора.

Рисунок 12.3.3. $ \ bf V $ — это проекция $ \ bf A $ на $ \ bf B $.

Обратите внимание, что фраза «проекция на $ \ bf B $» немного вводит в заблуждение. если понимать буквально; все, что дает $ \ bf B $, — это направление; то длина $ \ bf B $ не влияет на конечный вектор. В рисунок 12.3.4, например, $ \ bf B $ короче, чем вектор проекции, но это вполне приемлемо.

Рисунок 12.3.4. $ \ bf V $ — это проекция $ \ bf A $ на $ \ bf B $.

Пример 12.3,4 Физическая сила — это векторная величина. Часто бывает необходимо вычислить «компонент» силы, действующей в другом направлении, чем применяется сила. Например, предположим, что вес в десять фунтов равен опираясь на наклонную плоскость — например, на скатную крышу. Сила тяжести оказывает на объект силу в десять фунтов, направленную прямо вниз. Это полезно подумать о составляющей этой силы, направленной вниз и параллельно крыше, а компонент вниз и прямо в крыша. Эти силы являются проекциями вектора силы на векторы параллельны и перпендикулярны крыше.2} \ langle1, — \ sqrt3 \ rangle = {10 \ sqrt3 \ over 2} {\ langle1, — \ sqrt3 \ rangle \ over2} = \ langle 5 \ sqrt3 / 2, -15 / 2 \ rangle \ cr } $$ длиной $ \ ds 5 \ sqrt3 $. Таким образом, сила в 5 фунтов тянет объект. вниз по крыше, в то время как сила в $ \ ds 5 \ sqrt3 $ фунтов тянет объект в крышу. $ \ квадрат $

Рисунок 12.3.5. Компоненты силы.

У скалярного произведения есть некоторые знакомые свойства, которые будут пригодятся позже, поэтому мы перечислим их здесь. Это можно доказать, написав векторов в координатной форме с последующим выполнением указанных расчеты; впоследствии может быть проще использовать свойства вместо расчета с координатами.2 $

2. $ {\ bf u} \ cdot {\ bf v} = {\ bf v} \ cdot {\ bf u} $

3. $ {\ bf u} \ cdot ({\ bf v} + {\ bf w}) = {\ bf u} \ cdot {\ bf v} + {\ bf u} \ cdot {\ bf w} $

4. $ (a {\ bf u}) \ cdot {\ bf v} = a ({\ bf u} \ cdot {\ bf v}) = {\ bf u} \ cdot (a {\ bf v}) $

$ \ qed $

Упражнения 12.3

Вы можете использовать Sage для вычисления скалярных произведений и связанных величин, таких как скалярная и векторная проекции.

Пр. 12.3.1 Найдите $ \ langle 1,1,1 \ rangle \ cdot \ langle 2, -3,4 \ rangle $.(отвечать)

Пр. 12.3.2 Найдите $ \ langle 1,2,0 \ rangle \ cdot \ langle 0,0,57 \ rangle $. (отвечать)

Пр. 12.3.3 Найдите $ \ langle 3,2,1 \ rangle \ cdot \ langle 0,1,0 \ rangle $. (отвечать)

Пр. 12.3.4 Найдите $ \ langle -1, -2,5 \ rangle \ cdot \ langle 1,0, -1 \ rangle $. (отвечать)

Пример 12.3.5 Найдите $ \ langle 3,4,6 \ rangle \ cdot \ langle 2,3,4 \ rangle $. (отвечать)

Пр. 12.3.6 Найдите косинус угла между $ \ langle 1,2,3 \ rangle $ и $ \ langle 1,1,1 \ rangle $; при необходимости используйте калькулятор, чтобы найти угол.(отвечать)

Пример 12.3.7 Найдите косинус угла между $ \ langle -1, -2, -3 \ rangle $ и $ \ langle 5,0,2 \ rangle $; при необходимости используйте калькулятор, чтобы найти угол. (отвечать)

Пр. 12.3.8 Найдите косинус угла между $ \ langle 47,100,0 \ rangle $ и $ \ langle 0,0,5 \ rangle $; при необходимости используйте калькулятор, чтобы найти угол. (отвечать)

Пр. 12.3.9 Найдите косинус угла между $ \ langle 1,0,1 \ rangle $ и $ \ langle 0,1,1 \ rangle $; при необходимости используйте калькулятор, чтобы найти угол.(отвечать)

Пр. 12.3.10 Найдите косинус угла между $ \ langle 2,0,0 \ rangle $ и $ \ langle -1,1, -1 \ rangle $; при необходимости используйте калькулятор, чтобы найти угол. (отвечать)

Пр. 12.3.11 Найдите угол между диагональю куба и одним из ребра, прилегающие к диагонали. (отвечать)

Пр. 12.3.12 Найдите скалярную и векторную проекции $ \ langle 1,2,3 \ rangle $ на $ \ langle 1,2,0 \ rangle $. (отвечать)

Пр. 12.\ circ $ от горизонтали. Результирующая сила тянуть вагон по горизонтали земля должна быть 10 фунтов. Какова величина необходимого заставить $ {\ bf F} $? (отвечать)

Пр. 12.3.19 Используйте скалярное произведение, чтобы найти ненулевой вектор $ {\ bf w} $ перпендикулярно как $ {\ bf u} = \ langle 1,2, -3 \ rangle $, так и $ {\ bf v} = \ langle 2,0,1 \ rangle $. (отвечать)

Пр. 12.3.20 Пусть $ {\ bf x} = \ langle 1,1,0 \ rangle $ и $ {\ bf y} = \ langle 2,4,2 \ rangle $. Найдите единичный вектор, перпендикулярный обоим $ \ bf x $ и $ \ bf y $.

alexxlab

Добавить комментарий

Ваш адрес email не будет опубликован. Обязательные поля помечены *